Você está na página 1de 120

1 Analeptical remedy of reflective type from the H-cholinomimetics group

was given to the patient for restoration of breathing after poisoning with carbon
monoxide. What medicine was prescribed to the patient?
A Pentamin
B Lobeline hydrochloride *
C Adrenalin hydrochloride
D Mesaton
E Atropine sulphate
2 Patient with bronchial asthma was taking tablets which caused insomnia,
headache, increased blood pressure. What medecine can cause such
complications?
A Euphyline
B Adrenaline
C Ephedrine*
D Chromolin sodium
E Izadrine
3 An aged patient complains of headache, dizziness, quick tiredness,
worsening of memory. Anamnesis: craniocerebral injury. Medicine of what
group should be prescribed?
A Somnific
B Neuroleptics
C Nootropics*
D Sedatives
E Analgetics
4 Spasm of smooth muscle of bronchi developed in the patient. Usage of
activators of what membrane cytoreceptors is fisiologically valid to decrease
attack?
A М-cholinoreceptors
B α-аdrenoreceptors
C β-adrenoreceptors *
D α- and β-аdrenoreceptors
E Н-cholinoreceptors
5 Patient with complaints of dryness in the mouth, photophobia and vision
violation was admitted to the reception-room. Skin is hyperemic, dry, pupils
are dilated, tachycardia. Poisoning with belladonna alkaloids was diagnosed on
further examination. What medicine should be prescribed?
A Diazepam
B Armine
C Pilocarpine
D Prozerin*
E Dipyroxim
6 Patient in the unconscious state was admitted to the emergency room. Skin is
cold, pupils are delayed, breathing is heavy, with cycles of the Cheyne-Stokes
type, blood pressure is decreased, urinary bladder is overloaded. Poisoning with
what substance is the most likely?
A М-cholinergic antagonists
B Sedatives
C Narcotic analgesics *
D Non-narcotic analgesics
E None of the above
7 The patient has taken the mixture prescribed by neuropathologist for
neurasthenia for 2 weeks. Patient felt better but developed coryza,
conjunctivitis, rash, inertia, decrease of memory. Bromizm was diagnosed.
What should be prescribed to decrease symptoms?
A Glucose solution 5 %
B Asparcam
C Polyglucin
D Natrium chloride *
E All of the above
8 The patient was treated medically for psychosis for 2 weeks. Patient's
condition improved but rigidity, tremor, hypokinesia developed. Which of the
drugs can cause such complications?
A Sydnocarb
B Diphenine
C Imipramine
D Chlordiazepoxide
E Aminazine *
9 To anaesthetize the manipulation connected with burn surface treatment, a
patient was intravenously injected a medication for short-acting narcosis. 1
minute later the patient being under anaesthesia had elevated blood pressure,
tachycardia, increased tone of sceletal muscles; the reflexes were reserved.
After awakening the patient had desorientation and visual hallucinations. What
medication was injected to the patient?
A Nitrous oxide
B Ketamine*
C Thiopental sodium
D Sombrevin
E Diethylether
10 After the usage of acetylsalicylic acid a patient developed epigastric pain
because of exacerbation of his ulcer. What are the principles of this medication
ulcerogenity?
A Immunodepressive effect
B Cholagogic effect
C Antiprostaglandinic effect*
D Stimulation of pepsin secretion
E Spasm of vessels
11 A patient suffers from nocturnal paroxysms of bronchial asthma
accompanied by bradycardia, spastic intestinal pains and diarrhea. Medicine of
what group can releave these symptoms?
A Sympatholytics
B H-cholinergic receptors
C M-cholinergic receptors*
D Saluretics
E β-adrenoblockers
12 A patient with bronchial asthma is administered inhalation of 0,5 % isadrine
solution. Bronchospasm was releaved, but the patient began complaining of
pain in the heart region and palpitation. It is connected with the stimulation of:
A α1 –adrenoreceptors
B β1 –adrenoreceptors*
C β2 –adrenoreceptors
D Acetylcholine synthesis
E M-cholinergic receptors
13 A patient with fibrillation, who has had bronchial asthma in his anamnethis,
should be administered antiarrhythmic drug. Which preparation from the
suggested group is contraindicated for this patient?
A Ajmaline
B Nifedipine
C Anapriline (Propranolol)*
D Novocainamid
E Verapamil
14 A 25-year-old woman with red and itchy eczematoid dermatitis visits your
office. She had a dental procedure one day earlier with administration of a local
anesthetic. There were no other findings, although she indicated that she had a
history of allergic reactions. Which of the following drugs is most likely
involved?
A Procaine *
B Bupivacaine
C Lidocaine
D Etidocaine
E Cocaine
15 To anaesthetize the manipulation related to burn surface treatment, a patient
was intravenously injected a medication for short-acting narcosis. 1 minute later
the patient being under anaesthesia had increased blood pressure, tachycardia,
increased tone of sceletal muscles; reflexes remained. After awakening the
patient had desorientation and visual hallucinations. What medication was the
patient injected?
A Sombrevin
B Ketamine *
C Diethylether
D Thiopental sodium
E Nitrous oxide
16 A patient with bronchial asthma had been taking tablets which caused
insomnia, headache, increased blood pressure. What medecine can cause such
complications?
A Chromolin sodium
B Izadrine
C Euphyline
D Adrenaline
E Ephedrine *
17 A patient has been taking a mixture prescribed by neuropathologist for
neurasthenia for two weeks. The patient feels better but has developed coryza,
conjunctivitis, rash, inertia, decrease of memory. She is diagnosed with
bromizm. What should be prescribed to decrease the symptoms?
A Polyglucin
B Natrium chloride *
C Asparcam
D Glucose solution 5 %
E All of the above
18 A 13-year-old girl with history of asthma complained of cough, dyspnea and
wheezing. Her symptoms became so severe that her parents brought her to the
emergency room. Physical examination revealed diaphoresis, dyspnea,
tachycardia and tachypnea. Her respiratory rate was 42/min, pulse rate was 110
beats per minute, and blood pressure was 130/70 mm Hg. Choose from the
following list the most appropriate drug to reverse the bronchoconstriction
rapidly:
A Beclomethasone
B Ipratropium
C Methylprednidsolone
D Salbutamol *
E Cromolyn
19 A 42-year-old man who has been injured in a car accident is brought into the
emergency room. His blood alcohol level on admission is 250 mg/dL. Hospital
records show a prior hospitalization for alcohol related seizures. His wife
confirms that he has been drinking heavily for 3 weeks. What treatment should
be provided to the patient if he goes into withdrawal?
A Phenytoin
B Pentobarbital
C None of the above
D Phenobarbital
E Diazepam *
20 A 50-year-old male farm worker has been brought to the emergency room.
He was found confused in the orchard and since then has remained
unconscious. His heart rate is 45 and his blood pressure is 80/40 mm Hg. He is
sweating and salivating profusely. Which of the following should be
prescribed?
A Atropine sulfas*
B Proserine
C Physostigmine
D Norepinephrine
E Pentamine
21 A patient with complaints of dryness in the mouth, photophobia and vision
impairment was admitted to the reception-room. Skin is hyperemic, dry, pupils
are dilated, tachycardia. Poisoning with belladonna alkaloids was diagnosed on
further examination. What medicine should be prescribed
A Pilocarpine
B Dipiroxim
C Prozerin *
D Diazepam
E Armine
22 A 25-year-old woman with red and itchy eczematoid dermatitis visits your
office. She had a dental procedure one day earlier with administration of a local
anesthetic. There were no other findings, although she indicated that she had a
history of allergic reactions. Which of the following drugs is most likely
involved?
A Etidocaine
B Cocaine
C Bupivacaine
D Lidocaine
E Procaine *
23 A patient has got a spasm of smooth muscles of bronchi. Activators of what
membrane cytoreceptors are phisiologically reasoned to stop an attack?
A beta-adrenoreceptors*
B alpha- and beta –аdrenoreceptors
C Н-cholinoreceptors
D M-, H-cholinoreceptors
E М-cholinoreceptors
24 Introduction of a pharmaceutical substance to an experimental animal
resulted in reduction of salivation, pupil mydriasis. Next intravenous
introduction of acetylcholine didn't lead to any significant changes of heart rate.
Name this substance:
A Proserin
B Atropine*
C Propranolol
D Adrenaline
E Salbutamol
25 A patient with II stage hypertension has been taking one of hypotensive
medications for the purpose of treatment. After a time arterial pressure
decreased, but the patient started complaining of flaccidity, sleepiness,
indifference. A bit later he felt stomach pain. He was diagnosed with ulcer.
What hypotensive medication has the patient been taking?
A Captopril
B Dibazole
C Reserpine*
D Furosemide
E Verapamil
26 A patient had to go through an operation. Doctors introduced him dithylinum
(listenone) and performed intubation. After the end of operation and cessation
of anesthesia the independent respiration wasn't restored. Which enzyme deficit
prolongs the action of muscle relaxant?
A Psendocholinesterase*
B KiVaadenosine triphosphatase
C iVacetyltransferase
D Succinate dehydrogenase
E Carbanhydrase
27 A patient ill with bronchial asthma didn't inform his doctor that he had
attacks of stenocardia. Doctor administered him a medication, which taking
resulted in less frequent attacks of bronchial asthma, but stenocardia attacks
became more frequent. What medication was administered?
A Cromolyn sodium
B Salbutamol
C Aminophylline
D Phenotherol
E Isadrin*
28 A patient with fracture of his lower jaw was admitted to the maxillofacial
department. It was decided to fix his bones surgically under anaesthetic. After
intravenous introduction of muscle relaxant there arose short fibrillar
contractions of the patient's facial muscles. What muscle relaxant was applied?
A Dithylinum*
B Pipecuronium bromide
C Diazepam
D Melictine
E Tubocurarin chloride
29 Examination of a patient revealed extremely myotic pupils, sleepiness,
infrequent ChainStoke's respiration, urinary retention, slowingdown of heart
rate, enhancement of spinal reflexes. What substance caused the poisoning?
A Atropine sulfas
B Phosphactole
C Caffeine
D Morphine*
E Barbital
30 A patient who has been treated with diazepam on account of neurosis
complains of toothache. Doctor administered him an analgetic, but its dose was
lower than average therapeutic dose. What phenomenon did the doctor take into
account while prescribing the patient an underdose?
A Summation
B Potentiation*
C Cumulation
D Tolerance
E Drug dependence
31 On the 2-3rd day after stomach resection intestinal peristalsis wasn't restore
D. What is to be administered for stimulation of gastrointestinal tract?
A Atropine sulfate
B Proserin *
C Prasosin
D Cyclodole
E Acetylcholine
32 A stomatologist injected a patient with a certain drug in order to reduce
salivation during tooth filling. What drug is it?
A Atropine sulfate *
B Pilocarpine hydrochloride
C Proserin
D Mesaton
E Adrenaline hydrochloride
33 A patient with acute morphine poisoning was delivered to a hospital. What
specific narcotic antagonist should be chosen in this case?
A Unithiol
B Naloxone *
C Methacin
D Digoxin
E Paracetamol
34 A patient who has been treated in a neural clinic and has been taking a
sedative for a long time got the following complication: cough, rhinitis, epiphor
A. What drug caused these disturbances?
A Phenazepam
B Valerian
C Sodium bromide *
D Diazepam
E Reserpine
35 An ophthalmologist used a 1% mesaton solution for the diagnostic purpose
(pupil dilation for eye-ground examination). What is the cause of mydriasis
induced by the drug?
A Activation of 1 adrenoreceptors *
B Block of 1 adrenoreceptors
C Activation of M-cholinoreceptors
D Activation of 1adrenoreceptors
E Activation of 2adrenoreceptors
36 If a man has an attack of bronchiospasm it is necessary to reduce the effect
of vagus on smooth muscles of bronchi. What membrane cytoreceptors should
be blocked for this purpose?
A -adrenoreceptors
B -adrenoreceptors
C - and -adrenoreceptors
D N-cholinoreceptors
E M-cholinoreceptors *
37 A patient came to the hospital complaining about quick fatigability and
apparent muscle weakness. Examination revealed an autoimmune disease that
causes disorder of functional receptor condition in neuromuscular synapses.
What transmitter will be blocked?
A Dopamine
B Serotonin
C Glycine
D Acetylcholine *
E Noradrenalin
38 A patient with drug intoxication presented with the dryness of oral mucous
membrane and mydriatic pupils. Such action of this drug is associated with the
following effect:
A Adrenoreceptor block
B Muscarinic cholinoreceptor stumulation
C Adrenoreceptor stimulation
D Muscarinic cholinoreceptor block *
E Nicotinic cholinoreceptor stumulation
39 A patient consulted a physician about muscle rigidity, constrained
movements, permanent arm tremor. The patient was diagnosed with Parkinson's
diseas e. What preparation should be administered?
A Phenobarbital
B Levodopa *
C Diazepam
D Phenytoin
E Ethosuximide
40 A patient with myocardial infarction was admitted to the cardiological
department. For pain relief it was decided to potentiate fentanyl action with a
neurolepti c. Which of the following neuroleptics is the most suitable for
neuroleptanalgesia?
A Haloperidol
B Triftazine
C Aminazine
D Sulpiride
E Droperidol *
41 A patient with coronary artery disease was admitted to the cardiological
department. For stenocardia prevention a drug from the group of -adrenoceptor
blockers was administered. What drug is it?
A Morphine hydrochloride
B Oxytocin
C Metoprolol *
D Furosemide
E Atropine sulfate
42 A patient with a limb fracture must be administered a depolarizing drug from
the myorelaxant group for the purpose of a short-time surgery. What drug is it?
A Atropine sulfate
B Dithylinum *
C Tubocurarine chloride
D Cytitonum
E Pentaminum
43 A 63-year-old patient with collapse presentations was delivered to the
emergency hospital. A physician has chosen noradrenalin against hypotension.
What is its mechanism of action?
A Activation of 1-adrenoreceptors *
B Activation of serotonin receptors
C Activation of dopamine receptors
D Block of M-cholinoreceptors
E Activation of -adrenoreceptors
44 A patient suffering from myasthenia has been administered proserin. After
its administration the patient has got nausea, diarrhea, twitch of tongue and
skeletal muscles. What drug would help to eliminate the intoxication?
A Atropine sulfate *
B Pyridostigmine bromide
C Isadrine
D Mesatonum
E Physostigmine
45 A 45-year-old patient suffers from neurosis characterized by irritability,
sleeplessness, motiveless anxiety. What drug would eliminate all the
symptoms?
A Levodopa
B Pyracetam
C Valerian extract
D Diazepam *
E Caffeine sodium benzoate
46 Acetylcholine IS NOT a specific neurotransmitter at:
A Sympathetic ganglia
B Sympathetic postganglionic nerve endings*
C Parasympathetic ganglia
D Parasympathetic postganglionic nerve endings
E None of the above
47 Muscarinic receptors are located in:
A Autonomic ganglia
B Skeletal muscle neuromuscular junctions
C Autonomic effector cells*
D Sensory carotid sinus baroreceptor zone
E All of the above
48 Indicate the location of M2 cholinoreceptor type:
A Heart*
B Glands
C Smooth muscle
D Endothelium
E All of the above
49 The symptoms of mushroom poisoning include all of the following
EXCEPT:
A Salivation, lacrimation, nausea, vomiting
B Dryness of mouth, hyperpyrexia, hallucination*
C Headache, abdominal colic
D Bradycardia, hypotension and shock
E None of the above
50 Which of the following cholinomimetics activates both muscarinic and
nicotinic receptors?
A Lobeline
B Pilocarpine
C Nicotine
D Carbacholine*
E Adrenalin hydrochloride
51 Indicate a cholinomimetic agent, which is related to direct-acting drugs:
A Aceclidine
B Cytitone
C Carbacholine*
D Proserine
E Dipiroxim
52 Characteristics of carbacholine include all of the following EXCEPT:
A It decreases intraocular pressure
B It causes mydriasis*
C It exerts both nicotinic and muscarinic effects
D It is resistant to acethylcholiesterase
E All of the above
53 Acetylcholine IS NOT used in clinical practice because:
A It is very toxic
B The doses required are very high
C It is very rapidly hydrolyzed*
D It is very costly
E None of the above
54 Which of the following direct-acting cholinomimetics has the shortest
duration of action?
A Acetylcholine *
B Proserinum
C Galanthamini hydrobromidum
D Pridostigmini bromidum
E Cytitonum
55 A M-cholinimimetic agent is:
A Carbacholinum
B Pilocarpini hydrochloridum*
C Cytitonum
D Alloximum
E Atropini sulfas
56 Characteristics of pilocarpine include all of the following EXCEPT:
A It is a tertiary amine alkaloid
B It causes miosis and a decrease in intraocular pressure
C Causes a decrease in secretory and motor activity of gut*
D It is useful in the treatment of glaucoma
E All of the above
57 Which of the following cholinomimetics is a plant derivative with lower
potency than nicotine but with a similar spectrum of action?
A Lobelini hydrochloridum*
B Pilocarpini hydrochloridum
C Carbocholinum
D Aceclidinum
E Proserinum
58 Which of the following cholinomimetics is indirect-acting?
A Lobeline hydrochloridum
B Proserinum*
C Pilocarpini hydrochloridum
D Carbocholinum
E Atropini sulfas
59 The mechanism of action of indirect-acting cholinomimetic agents is:
A Binding to and activation of muscarinic or nicotinic receptors
B Inhibition of the hydrolysis of endogenous acetylcholine *
C Stimulation of the action of acetylcholinesterase
D Releasing acetylcholine from storage sites
E All of the above
60 Indicate a reversible cholinesterase inhibitor:
A Proserinum*
B Carbacholinum
C Pilocarpini hydrochloridum
D Phosphacolum
E Cytitonum
61 Which of the following cholinesterase inhibitors is irreversible?
A Carbacholinum
B Pilocarpini hydrochloridum
C Proserinum
D Phosphacolum*
E Cytitonum
62 Indicate M-cholinomimetic:
A Carbocholinum
B Pilocarpini hydrochloridum*
C Proserinum
D Galanthamini hydrobromidum
E Pyridostigmini bromidum
63 Proserinum increases all of the following effects EXCEPT:
A Lacrimation
B Bronchodilation *
C Muscle twitching
D Salivation
E Bradycardia
64 Сholinesterase inhibitors do not produce:
A Bradycardia, no change or modest fall in blood pressure
B Increased strength of muscle contraction, especially in muscles
weakened by myasthenia gravis
C Miosis and reduction of intraocular pressure
D Dramatic hypertension and tachycardia*
E All of the above
65 Which of the following cholinomimetics is commonly used in the treatment
of glaucoma?
A Pilocarpini hydrochloridum*
B Lobelini hydrochloridum
C Carbocholinum
D Atropini sulfas
E Galanthamini hydrobromidum
66 Which of the following cholinomimetic is most widely used for paralytic
ileus and atony of the urinary bladder?
A Pilocarpini hydrochloridum
B Lobelini hydrochloridum
C Proserinum*
D Atropini sulfas
E Phosphacolum
67 . Chronic long-term therapy of myasthenia is usually accomplished with:
A Pilocarpini hydrochloridum
B Lobelini hydrochloridum
C Proserinum*
D Atropini sulfas
E Phosphacolum
68 Which of the following cholinomimetics is a drug of choice for reversing the
effects of nondepolarizing neuromuscular relaxants?
A Pilocarpini hydrochloridum
B Lobelini hydrochloridum
C Proserinum*
D Atropini sulfas
E Phosphacolum
69 Indicate the reversible cholinesterase inhibitor, which penetrates the blood-
brain barrier:
A Pilocarpini hydrochloridum
B Proserinum
C Carbocholinum
D Atropini sulfas
E Galanthamini hydrobromidum*
70 Which of the following cholinomimetics is used in the treatment of atropine
intoxication?
A Pilocarpini hydrochloridum
B Aceclidinum
C Carbocholinum
D Proserinum*
E Phosphacolum
71 The symptoms of excessive stimulation of muscarinic receptors include all
of the following EXCEPT:
A Abdominal cramps, diarrhea
B Increased salivation, excessive bronchial secretion
C Miosis
D Bradycardia
E Weakness of all skeletal muscles*
72 The excessive stimulation of muscarinic receptors by pilocarpine and
choline esters is blocked competitively by:
A Pilocarpini hydrochloridum
B Aceclidinum
C Carbocholinum
D Proserinum
E Atropini sulfas*
73 The toxic effects of a large dose of nicotine include all of the following
EXCEPT:
A Hypotension and bradycardia*
B Convulsions, coma
C Respiratory arrest
D Skeletal muscle depolarization blockade and respiratory paralysis
E Hypertension and cardiac arrhythmias
74 The dominant initial sights of acute cholinesterase inhibitors intoxication
include all of the following EXCEPT:
A Salivation
B Sweating
C Mydriasis*
D Bronchial constriction
E Vomiting and diarrhea
75 Which of the following drugs is used for acute toxic effects of M-
cholinomimetics?
A Atropini sulfas*
B Proserinum
C Galanthamini hydrobromidum
D Carbocholinum
E Proserinum
76 The group of nicotinic receptor-blocking drugs consists of:
A Ganglion-blockers
B Atropine-similar drugs
C Neuromuscular junction blockers
D Anticholinesterases
E Ganglion-blockers and Neuromuscular junction blockers*
77 Which of the following drugs is both a muscarinic and nicotinic blocker?
A Atropini sulfas
B Cyclodolum*
C Galanthamini hydrobromidum
D Phosphacolum
E Pirilenum
78 Indicate a muscarinic receptor-blocking drug:
A Scopolamini hydrobromidum*
B Proserinum
C Galanthamini hydrobromidum
D Phosphacolum
E Pirilenum
79 Which of the following agents is a ganglion-blocking drug?
A Scopolamini hydrobromidum
B Proserinum
C Galanthamini hydrobromidum
D Phosphacolum
E Pirilenum*
80 Indicate the skeletal muscle relaxant, which is a depolarizing agent:
A Dithyllinum*
B Pipecuronii bromidum
C Mellictinum
D Tubocurarini chloridum
E Amizylum
81 Which of the following drugs is a nondepolarizing muscle relaxant?
A Dithyllinum
B Pirilenum
C Aceclidinum
D Tubocurarini chloridum*
E Amizylum
82 Indicate the drug, which is rapidly and fully distributed into CNS and has a
greater effect than most other antimuscarinic agents?
A Atropini sulfas
B Scopolamini hydrobromidum*
C Proserinum
D Phosphacolum
E Carbocholinum
83 The effect of the drug on parasympathetic function declines rapidly in all
organs EXCEPT:
A Eye*
B Heart
C Smooth muscle organs
D Glands
E All of the above
84 The mechanism of Atropini sulfas action is:
A Competitive ganglion blockade
B Competitive muscarinic blockade*
C Competitive neuromuscular blockade
D Noncompetitive neuromuscular blockade
E All of the above
85 Atropini sulfas is highly selective for:
A M1 receptor subtype
B M2 receptor subtype
C M3 receptor subtype
D All of the above*
E None of the above
86 Which of the following antimuscarinic drugs is often effective in preventing
or reversing vestibular disturbances, especially motion sickness?
A Atropini sulfas
B Scopolamini hydrobromidum*
C Proserinum
D Phosphacolum
E Carbocholinum
87 Atropini sulfas causes:
A Miosis, a reduction in intraocular pressure and cyclospasm
B Mydriasis, a rise in intraocular pressure and cycloplegia*
C Miosis, a rise in intraocular pressure and cycloplegia
D Mydriasis, a rise in intraocular pressure and cyclospasm
E None of the above
88 Patients complain on dry or “sandy” eyes when receiving large doses of:
A Atropini sulfas*
B Proserinum
C Phosphacolum
D Carbocholinum
E Galanthamini hydrobromidum
89 Atropini sulfas causes:
A Bradycardia, hypotension and bronchoconstriction
B Tachycardia, little effect on blood pressure and bronchodilation*
C Decrease in contractile strength, conduction velocity through the AV
node
D Tachycardia, hypertensive crisis and bronchodilation
E None of the above
90 Atropine is frequently used prior to administration of inhalant anesthetics to
reduce:
A Muscle tone
B Secretions*
C Nausea and vomiting
D All of the above
E None of the above
91 Atropine is now rarely used for the treatment of peptic ulcer because of:
A Slow gastric empting and prolongation of the exposure of the ulcer
bed to acid
B Low efficiency and necessity of large doses
C Adverse effects
D All of the above*
E None of the above
92 Which of the following antimuscarinic drugs is a selective M1 blocker?
A Atropini sulfas
B Scopolamini hydrobromidum
C Pirenzepinum*
D Benzohexonium
E Hygronium
93 Atropini sulfas causes:
A Spasmolitic activity*
B Intestinal hypermotility
C Stimulation of contraction in the gut
D Stimulation of secretory activity
E None of the above
94 Which of the following drugs is useful in the treatment of uterine spasms?
A Proserinum
B Phosphacolum
C Carbocholinum
D Atropini sulfas*
E Galanthamini hydrobromidum
95 The pharmacologic actions of Scopolamini hydrobromidum most closely
resemble those of:
A Proserinum
B Atropini sulfas*
C Phosphacolum
D Carbocholinum
E Pilocarpini hydrochloridum
96 Compared with Atropini sulfas, Scopolamini hydrobromidum has all of the
following properties EXCEPT:
A More marked central effect
B Less potent in decreasing bronchial, salivary and sweat gland
secretion*
C More potent in producing mydriasis and cycloplegia
D Lower effects on the heart, bronchial muscle and intestines
E None of the above
97 Which of the following drugs is useful in the treatment of Parkinson′s
disease?
A Amizylum*
B Proserinum
C Atropini sulfas
D Phosphacolum
E Benzohexonium
98 Which of the following agents is used as an inhalation drug in asthma?
A Atropini sulfas
B Ipratropium bromide*
C Lobelini hydrochloridum
D Amizylum
E Galanthamini hydrobromidum
99 Indicate an antimuscarinic drug, which is effective in the treatment of
mushroom poising:
A Atropini sulfas*
B Proserinum
C Pilocarpini hydrochloridum
D Carbocholinum
E Aceclidinum
100 Antimuscarinics are used in the treatment of the following disorders
EXCEPT:
A Motion sickness
B Glaucoma*
C Hyperhidrosis
D Asthma
E Intestinal, renal and hepatic colic
101 The Atropini sulfas poisoning includes all of the following symptoms
EXCEPT:
A Mydriasis, cycloplegia
B Hyperthermia, hot and flushed skin
C Dry mouth
D Agitation and delirium
E Bradicardia, orthostatic hypotension*
102 Contraindications to the use of antimuscarinic drugs are all of the
following EXCEPT:
A Glaucoma
B Myasthenia
C Bronchial asthma*
D Paralytic ileus and atony of the urinary bladder
E All of the above
103 The applications of the ganglion blockers have disappeared because of
all of the following reasons EXCEPT:
A Orthostatic hypotension
B Lack of selectivity
C Homeostatic reflexes block
D Respiratory depression*
E None of the above
104 Which of the following agents is a short-acting ganglion blocker?
A Hygronium*
B Pirilenum
C Benzohexonium
D Pentaminum
E Mellictinum
105 Ganglion blocking drugs are used for the following emergencies
EXCEPT:
A Hypertensive crises
B Controlled hypotension
C Cardiovascular collapse*
D Pulmonary edema
E Peripheral vascular spasms
106 Agents that produce neuromuscular blockade act by inhibiting:
A Interaction of acetylcholine with cholinergic receptors*
B Release of acetylcholine from prejunctional membrane
C Packaging of acetylcholine into synaptic vesicles
D Reuptake of acetylcholine into the nerve ending
E All of the above
107 Skeletal muscle relaxation and paralysis can occur from interruption
of functions at several sites, including all of the following EXCEPT:
A Nicotinic acethylcholine receptors
B Muscarinic acethylcholine receptors*
C The motor end plate
D Contractile apparatus
E None of the above
108 Nondepolarisation neuromuscular blocking agents:
A Block acetylcholine reuptake
B Prevent access of the transmitter to its receptor and depolarization*
C Block transmission by an excess of a depolarizing agonist
D All of the above
E None of the above
109 Indicate the long-acting neuromuscular blocking agent:
A Tubocurarini chloridum*
B Mellictinum
C Dithylinum
D Cyclodolum
E Amizylum
110 Which of the following neuromuscular blocking drugs is an
intermediate-duration muscle relaxant?
A Tubocurarini chloridum
B Mellictinum*
C Dithylinum
D Pipecuronium bromidum
E Amizylum
111 Depolarizing agents include all of the following properties EXCEPT:
A Interact with nicotinic receptor to compete with acetylcholine without
receptor activation*
B React with the nicotinic receptor to open the channel and cause
depolarisation of the end plate
C Cause desensitization, noncompetive block manifested by flaccid
paralysis
D Cholinesterase inhibitors do not have the ability to reverse the
blockade
E All of the above
112 Indicate muscles, which are more resistant to block and recover more
rapidly:
A Hand
B Leg
C Neck
D Diaphragm*
E All of the above
113 Which neuromuscular blocking agent has the potential to cause the
greatest release of histamine?
A Tubocurarini chloridum*
B Cyclodolum
C Amizylum
D All of the above
E None of the above
114 Effects seen only with depolarizing blockade include all of the
following EXCEPT:
A Hypercaliemia
B A decrease in intraocular pressure*
C Emesis
D Muscle pain
E All of the above
115 Which cholinergic blocking agent is contraindicated in patients with
glaucoma?
A Proserinum
B Atropini sulfas*
C Aceclidinum
D Pilocarpini hydrochloridum
E Carbocholinum
116 Indicate the following neuromuscular blocker, which would be
contraindicated in patients with renal failure:
A Pipecuronium bromidum*
B Aceclidinum
C Pilocarpini hydrochloridum
D Atropini sulfas
E Aceclidinum
117 Compared to ranitidine, cimetidine:
A is more effective in healing duodenal ulcers
B is less selective for H2 receptors
C produces a lower rate of recurrence of ulcers after cessation of
treatment
D is a more potent inhibitor of the hepatic P-450 system *
E has little tendency to produce antiandrogenic effects
118 In the treatment of gastric and duodenal ulcers:
A effective initial therapy may utilize antacids, sucralfate, or famotidine
*
B recurrence of ulcers happens in less than 10% of patients upon
cessation of therapy with H2 antagonists
C omeprazole should be reserved for maintenance therapy after 8 weeks
of treatment with ranitidine
D cimetidine is the drug of choice in older patients with impaired renal
function
E the use of ranitidine should not exceed 8 weeks in order to avoid
colonization of the stomach by bacteria
119 All of the following statements about misoprostol are true EXCEPT:
A inhibits acid secretion by parietal cells
B when given to patients taking NSAIDs, tends to replace endogenous
PGE2
C causes a significant incidence of abdominal pain and diarrhea
D is more effective than famotidine in healing gastric ulcers *
E can induce contraction of the uterus
120 Which of the following statements about omeprazole is the LEAST
valid?
A is indicated for the therapy of Zollinger-Ellison syndrome
B has a duration of action that far outlasts its lifetime in plasma
C formation of active drug requires low pH
D causes a greater inhibition of gastric acid secretion than does
ranitidine
E heals ulcers more slowly than cimetidine does *
121 Cisapride:
A inhibits the actions of dopamine in the G.I. tract
B can cause hyperprolactinemia
C promotes gastric emptying by stimulating 5HT 4 receptors in the
mesenteric plexus and other intramural ganglia *
D can be used to treat esophageal reflux in part because it reduces gastric
acidity as effectively as does cimetidine
E produces little or no increase in motility of the colon
122 Antacids that are used in the treatment of duodenal ulcers:
A are less efficacious than ranitidine in promoting healing of ulcers
B are usually mixtures of Al(OH)3 and Mg(OH)2 which neutralize H+ *
C are effective in part because they suppress gastric acid secretion
D are more apt to cause constipation as the dosage is raised
E have important bactericidal effects on Helicobacter pylori
123 Methylcellulose, diphenoxylate, bismuth subcitrate, bisacodyl,
MgSO4:
A Organic stool softener; promotes colonic motility *
B Poorly absorbed; used for rapid effect before bowel surgery
C Poorly absorbed opioid; slows transit of intestinal contents
D Promotes healing of peptic ulcers
E Promotes secretion of H2O into the colon; chronic use can produce
"cathartic colon"
124 Bismuth subcitrate, bisacodyl, MgSO4, methylcellulose,
diphenoxylate:
A Organic stool softener; promotes colonic motility
B Poorly absorbed; used for rapid effect before bowel surgery
C Poorly absorbed opioid; slows transit of intestinal contents
D Promotes healing of peptic ulcers *
E Promotes secretion of H2O into the colon; chronic use can produce
"cathartic colon"
125 Choose the substance that inhibits histamine induced gastric acid
secretion:
A PGI2 *
B Pyrilamine
C Diphenhydramine
D Meclizine
E Astemizole
126 Infections with Helicobacter pylori:
A can be suppressed by ranitidine but not by antacids
B can be eradicated with omeprazole and tetracycline
C can be exacerbated by treatment with misoprostol because the drug
stimulates bowel function
D cause the great majority of duodenal ulcers *
E can be suppressed by sucralfate
127 Which of the following diuretic agents would be the LEAST apt to
cause hypokalemic alkalosis?
A Hydrochlorothiazide
B triamterene *
C furosemide
D chlorthalidone
E bumetanide
128 Both sucrafate and bismuth subcitrate:
A eradicate Helicobacter pylori infections in the stomach
B form a protective barrier on ulcer craters*
C have significant acid neutralizing capacity
D have significant H2blocking actions
E can produce Al3+ toxicity in renally impaired patients
129 The mechanism by which thiazides induce hypokalemic alkalosis
includes all of the following steps except:
A increased delivery of Na to the late distal/cortical collecting duct
B increased intraluminal secretion of K by the principal cells
C inhibition of the NaCl cotransporter in the early distal tubule
D decreased concentration of Ca in the lumen of the cortical collecting
duct
E reduced proximal reabsorption of HCO3 *
130 Toxicities associated with the loop diuretics include all of the
following EXCEPT:
A Hypokalemia
B severe hyponatremia *
C hyperglycemia
D hypomagnesaemia
E hyperuricemia
131 Carbonic anhydrase inhibitors are not used to:
A treat glaucoma
B reduce the likelihood of renal stones *
C treat metabolic alkalosis
D promote the excretion of uric acid
E alkalinize the urine
132 Furosemide:
A reaches its site of action primarily by tubular secretion *
B produces alkalosis primarily by reducing secretion of H+ in the distal
tubule
C inhibits Na+ reabsorption in the proximal tubule
D reduces excretion of Ca2+ and Mg2+
E reduces renin secretion
133 Side effect associated with spironolactone but not with thiazide
diuretics:
A hypokalemia
B increased renin release
C gynecomastia *
D hyperuricemia
E hyperglycemia
134 Mannitol:
A is useful in the treatment of patients with congestive heart failure
B reaches its site of action largely by tubular secretion
C competes for tubular reabsorption of glucose
D can protect against accumulation of nephrotoxic substances *
E has no effect on the excretion of Na+
135 All of the following increase urinary excretion of Cl- EXCEPT:
A acetazolamide *
B furosemide
C mannitol
D ethacrynic acid
E amiloride
136 Which of the following statements is true for BOTH furosemide and
hydrochlorothiazide?
A promotes excretion of uric acid
B acutely decreases HCO3- excretion and can produce alkalosis with
chronic administration*
C promotes excretion of Ca2+
D reduces the osmotic gradient in the renal medulla
E reaches its site of action mostly by glomerular filtration
137 Which one of the following processes occurs in the loop of Henle?
A Thiazide diuretics inhibit Na+ reabsorption via ENaC (the
electrogenic Na+ channel in the late distal tubule and cortical collecting duct)
B Tubular (luminal) fluid becomes isotonic to plasma by the end of the
TAL.
C NH4+ is secreted in the net sense into the lumen.
D The lumen is electrically positive with respect to the interstitial fluid.
*
E Mg2+ is reabsorbed via an ATP powered pump in the tight junction
138 All of the following statements about metoclopramide are true
EXCEPT:
A can produce hyperprolactinemia
B often produces constipation *
C increases tone of the lower esophageal sphincter
D attenuates nausea produced by cancer chemotherapeutic agents
E actions on gastric motility are reduced by atropine
139 A 23-year-old graduate student goes on a cruise to the Bahamas. She
considers taking a drug for sea-sickness as she is prone to developing motion
sickness. Which compound and time of administration would be most effective?
A Transdermal scopolamine taken during the voyage
B Fexofenadine taken 4 hours prior to sailing
C Transdermal scopolamine taken 4 hours prior to sailing*
D Cetirizine taken 1 hour prior to sailing
E Metoclopramide taken during the voyage
140 Which of the following antiemetic drugs possesses clinically
significant dopaminergic antagonist activity, but is not used as an antipsychotic
agent?
A Ondansetron
B Promethazine
C Chlorpromazine
D Metoclopramide *
E Dimenhydrinate
141 Analeptical remedy of reflective type from the H-cholinomimetics
group was given to the patient for restoration of breathing after poisoning with
carbon monoxide. What medicine was prescribed to the patient?
A Pentamin
B Lobeline hydrochloride *
C Adrenaline hydrochloride
D Mesaton
E Atropine sulphate
142 Patient with bronchial asthma was taking tablets which caused
insomnia, headache, increased blood pressure. What medecine can cause such
complications?
A Euphyline
B Adrenaline
C Ephedrine*
D Chromolin sodium
E Izadrine
143 An aged patient complains of headache, dizziness, quick tiredness,
worsening of memory. Anamnesis: craniocerebral injury. Medicine of what
group should be prescribed?
A Somnific
B Neuroleptics
C Nootropics*
D Sedatives
E Analgetics
144 Spasm of smooth muscle of bronchi developed in the patient. Usage of
activators of what membrane cytoreceptors is fisiologically valid to decrease
attack?
A М-cholinoreceptors
B alpha-аdrenoreceptors
C beta-adrenoreceptors *
D alpha and beta-аdrenoreceptors
E Н-cholinoreceptors
145 Patient with complaints of dryness in the mouth, photophobia and
vision violation was admitted to the reception-room. Skin is hyperemic, dry,
pupils are dilated, tachycardia. Poisoning with belladonna alkaloids was
diagnosed on further examination. What medicine should be prescribed?
A Diazepam
B Armine
C Pilocarpine
D Prozerin*
E Dipyroxim
146 Patient in the unconscious state was admitted to the emergency room.
Skin is cold, pupils are delayed, breathing is heavy, with cycles of the Cheyne-
Stokes type, blood pressure is decreased, urinary bladder is overloaded.
Poisoning with what substance is the most likely?
A М-cholinergic antagonists
B Sedatives
C Narcotic analgesics *
D Non-narcotic analgesics
E None of the above
147 The patient has taken the mixture prescribed by neuropathologist for
neurasthenia for 2 weeks. Patient felt better but developed coryza,
conjunctivitis, rash, inertia, decrease of memory. Bromizm was diagnosed.
What should be prescribed to decrease symptoms?
A Glucose solution 5 %
B Asparcam
C Polyglucin
D Natrium chloride *
E All of the above
148 The patient was treated medically for psychosis for 2 weeks. Patient's
condition improved but rigidity, tremor, hypokinesia developed. Which of the
drugs can cause such complications?
A Sydnocarb
B Diphenine
C Imipramine
D Chlordiazepoxide
E Aminazine *
149 To anaesthetize the manipulation connected with burn surface
treatment, a patient was intravenously injected a medication for short-acting
narcosis. 1 minute later the patient being under anaesthesia had elevated blood
pressure, tachycardia, increased tone of sceletal muscles; the reflexes were
reserved. After awakening the patient had desorientation and visual
hallucinations. What medication was injected to the patient?
A Nitrous oxide
B Ketamine*
C Thiopental sodium
D Sombrevin
E Diethylether
150 After the usage of acetylsalicylic acid a patient developed epigastric
pain because of exacerbation of his ulcer. What are the principles of this
medication ulcerogenity?
A Immunodepressive effect
B Cholagogic effect
C Antiprostaglandinic effect*
D Stimulation of pepsin secretion
E Spasm of vessels
151 A patient suffers from nocturnal paroxysms of bronchial asthma
accompanied by bradycardia, spastic intestinal pains and diarrhea. Medicine of
what group can releave these symptoms?
A Sympatholytics
B H-cholinergic receptors
C M-cholinergic receptors*
D Saluretics
E β-adrenoblockers
152 A patient with bronchial asthma is administered inhalation of 0,5 %
isadrine solution. Bronchospasm was releaved, but the patient began
complaining of pain in the heart region and palpitation. It is connected with the
stimulation of:
A alpha1 –adrenoreceptors
B beta1 –adrenoreceptors*
C beta2 –adrenoreceptors
D Acetylcholine synthesis
E M-cholinergic receptors
153 A patient with fibrillation, who has had bronchial asthma in his
anamnethis, should be administered antiarrhythmic drug. Which preparation
from the suggested group is contraindicated for this patient?
A Ajmaline
B Nifedipine
C Anapriline (Propranolol)*
D Novocainamid
E Verapamil
154 A 25-year-old woman with red and itchy eczematoid dermatitis visits
your office. She had a dental procedure one day earlier with administration of a
local anesthetic. There were no other findings, although she indicated that she
had a history of allergic reactions. Which of the following drugs is most likely
involved?
A Procaine *
B Bupivacaine
C Lidocaine
D Etidocaine
E Cocaine
155 To anaesthetize the manipulation related to burn surface treatment, a
patient was intravenously injected a medication for short-acting narcosis. 1
minute later the patient being under anaesthesia had increased blood pressure,
tachycardia, increased tone of sceletal muscles; reflexes remained. After
awakening the patient had desorientation and visual hallucinations. What
medication was the patient injected?
A Sombrevin
B Ketamine *
C Diethylether
D Thiopental sodium
E Nitrous oxide
156 A patient with bronchial asthma had been taking tablets which caused
insomnia, headache, increased blood pressure. What medecine can cause such
complications?
A Chromolin sodium
B Izadrine
C Euphyline
D Adrenaline
E Ephedrine *
157 A patient has been taking a mixture prescribed by neuropathologist for
neurasthenia for two weeks. The patient feels better but has developed coryza,
conjunctivitis, rash, inertia, decrease of memory. She is diagnosed with
bromizm. What should be prescribed to decrease the symptoms?
A Polyglucin
B Natrium chloride *
C Asparcam
D Glucose solution 5 %
E All of the above
158 A 13-year-old girl with history of asthma complained of cough,
dyspnea and wheezing. Her symptoms became so severe that her parents
brought her to the emergency room. Physical examination revealed diaphoresis,
dyspnea, tachycardia and tachypnea. Her respiratory rate was 42/min, pulse rate
was 110 beats per minute, and blood pressure was 130/70 mm Hg. Choose from
the following list the most appropriate drug to reverse the bronchoconstriction
rapidly:
A Beclomethasone
B Ipratropium
C Methylprednidsolone
D Salbutamol *
E Cromolyn
159 A 42-year-old man who has been injured in a car accident is brought
into the emergency room. His blood alcohol level on admission is 250 mg/dL.
Hospital records show a prior hospitalization for alcohol related seizures. His
wife confirms that he has been drinking heavily for 3 weeks. What treatment
should be provided to the patient if he goes into withdrawal?
A Phenytoin
B Pentobarbital
C None of the above
D Phenobarbital
E Diazepam *
160 A 50-year-old male farm worker has been brought to the emergency
room. He was found confused in the orchard and since then has remained
unconscious. His heart rate is 45 and his blood pressure is 80/40 mm Hg. He is
sweating and salivating profusely. Which of the following should be
prescribed?
A Atropine sulfas*
B Proserine
C Physostigmine
D Norepinephrine
E Pentamine
161 A patient with complaints of dryness in the mouth, photophobia and
vision impairment was admitted to the reception-room. Skin is hyperemic, dry,
pupils are dilated, tachycardia. Poisoning with belladonna alkaloids was
diagnosed on further examination. What medicine should be prescribed
A Pilocarpine
B Dipiroxim
C Prozerin *
D Diazepam
E Armine
162 A 25-year-old woman with red and itchy eczematoid dermatitis visits
your office. She had a dental procedure one day earlier with administration of a
local anesthetic. There were no other findings, although she indicated that she
had a history of allergic reactions. Which of the following drugs is most likely
involved?
A Etidocaine
B Cocaine
C Bupivacaine
D Lidocaine
E Procaine *
163 A patient has got a spasm of smooth muscles of bronchi. Activators of
what membrane cytoreceptors are phisiologically reasoned to stop an attack?
A beta-adrenoreceptors*
B alpha- and beta –аdrenoreceptors
C Н-cholinoreceptors
D M-, H-cholinoreceptors
E М-cholinoreceptors
164 Introduction of a pharmaceutical substance to an experimental animal
resulted in reduction of salivation, pupil mydriasis. Next intravenous
introduction of acetylcholine didn't lead to any significant changes of heart rate.
Name this substance:
A Proserin
B Atropine*
C Propranolol
D Adrenaline
E Salbutamol
165 A patient with II stage hypertension has been taking one of
hypotensive medications for the purpose of treatment. After a time arterial
pressure decreased, but the patient started complaining of flaccidity, sleepiness,
indifference. A bit later he felt stomach pain. He was diagnosed with ulcer.
What hypotensive medication has the patient been taking?
A Captopril
B Dibazole
C Reserpine*
D Furosemide
E Verapamil
166 A patient had to go through an operation. Doctors introduced him
dithylinum (listenone) and performed intubation. After the end of operation and
cessation of anesthesia the independent respiration wasn't restored. Which
enzyme deficit prolongs the action of muscle relaxant?
A Psendocholinesterase*
B KiVaadenosine triphosphatase
C iVacetyltransferase
D Succinate dehydrogenase
E Carbanhydrase
167 A patient ill with bronchial asthma didn't inform his doctor that he had
attacks of stenocardia. Doctor administered him a medication, which taking
resulted in less frequent attacks of bronchial asthma, but stenocardia attacks
became more frequent. What medication was administered?
A Cromolyn sodium
B Salbutamol
C Aminophylline
D Phenotherol
E Isadrin*
168 A patient with fracture of his lower jaw was admitted to the
maxillofacial department. It was decided to fix his bones surgically under
anaesthetic. After intravenous introduction of muscle relaxant there arose short
fibrillar contractions of the patient's facial muscles. What muscle relaxant was
applied?
A Dithylinum*
B Pipecuronium bromide
C Diazepam
D Melictine
E Tubocurarin chloride
169 Examination of a patient revealed extremely myotic pupils, sleepiness,
infrequent ChainStoke's respiration, urinary retention, slowingdown of heart
rate, enhancement of spinal reflexes. What substance caused the poisoning?
A Atropine sulfas
B Phosphactole
C Caffeine
D Morphine*
E Barbital
170 A patient who has been treated with diazepam on account of neurosis
complains of toothache. Doctor administered him an analgetic, but its dose was
lower than average therapeutic dose. What phenomenon did the doctor take into
account while prescribing the patient an underdose?
A Summation
B Potentiation*
C Cumulation
D Tolerance
E Drug dependence
171 On the 2-3rd day after stomach resection intestinal peristalsis wasn't
restore D. What is to be administered for stimulation of gastrointestinal tract?
A Atropine sulfate
B Proserin *
C Prasosin
D Cyclodole
E Acetylcholine
172 A stomatologist injected a patient with a certain drug in order to
reduce salivation during tooth filling. What drug is it?
A Atropine sulfate *
B Pilocarpine hydrochloride
C Proserine
D Mesaton
E Adrenaline hydrochloride
173 A patient with acute morphine poisoning was delivered to a hospital.
What specific narcotic antagonist should be chosen in this case?
A Unithiol
B Naloxone *
C Methacin
D Digoxin
E Paracetamol
174 A patient who has been treated in a neural clinic and has been taking a
sedative for a long time got the following complication: cough, rhinitis, epiphor
A. What drug caused these disturbances?
A Phenazepam
B Valerian
C Sodium bromide *
D Diazepam
E Reserpine
175 An ophthalmologist used a 1% mesaton solution for the diagnostic
purpose (pupil dilation for eye-ground examination). What is the cause of
mydriasis induced by the drug?
A Activation of alpha1 adrenoreceptors *
B Block of alpha1 adrenoreceptors
C Activation of M-cholinoreceptors
D Activation of beta1adrenoreceptors
E Activation of alpha2adrenoreceptors
176 If a man has an attack of bronchiospasm it is necessary to reduce the
effect of vagus on smooth muscles of bronchi. What membrane cytoreceptors
should be blocked for this purpose?
A alpha-adrenoreceptors
B beta-adrenoreceptors
C alpha- and beta-adrenoreceptors
D N-cholinoreceptors
E M-cholinoreceptors *
177 A patient came to the hospital complaining about quick fatigability
and apparent muscle weakness. Examination revealed an autoimmune disease
that causes disorder of functional receptor condition in neuromuscular synapses.
What transmitter will be blocked?
A Dopamine
B Serotonin
C Glycine
D Acetylcholine *
E Noradrenalin
178 A patient with drug intoxication presented with the dryness of oral
mucous membrane and mydriatic pupils. Such action of this drug is associated
with the following effect:
A Adrenoreceptor block
B Muscarinic cholinoreceptor stumulation
C Adrenoreceptor stimulation
D Muscarinic cholinoreceptor block *
E Nicotinic cholinoreceptor stumulation
179 A patient consulted a physician about muscle rigidity, constrained
movements, permanent arm tremor. The patient was diagnosed with Parkinson's
diseas e. What preparation should be administered?
A Phenobarbital
B Levodopa *
C Diazepam
D Phenytoin
E Ethosuximide
180 A patient with myocardial infarction was admitted to the cardiological
department. For pain relief it was decided to potentiate fentanyl action with a
neurolepti c. Which of the following neuroleptics is the most suitable for
neuroleptanalgesia?
A Haloperidol
B Triftazine
C Aminazine
D Sulpiride
E Droperidol *
181 A patient with coronary artery disease was admitted to the
cardiological department. For stenocardia prevention a drug from the group of -
adrenoceptor blockers was administered. What drug is it?
A Morphine hydrochloride
B Oxytocin
C Metoprolol *
D Furosemide
E Atropine sulfate
182 A patient with a limb fracture must be administered a depolarizing
drug from the myorelaxant group for the purpose of a short-time surgery. What
drug is it?
A Atropine sulfate
B Dithylinum *
C Tubocurarine chloride
D Cytitonum
E Pentaminum
183 A 63-year-old patient with collapse presentations was delivered to the
emergency hospital. A physician has chosen noradrenalin against hypotension.
What is its mechanism of action?
A Activation of 1-adrenoreceptors *
B Activation of serotonin receptors
C Activation of dopamine receptors
D Block of M-cholinoreceptors
E Activation of -adrenoreceptors
184 A patient suffering from myasthenia has been administered proserin.
After its administration the patient has got nausea, diarrhea, twitch of tongue
and skeletal muscles. What drug would help to eliminate the intoxication?
A Atropine sulfate *
B Pyridostigmine bromide
C Isadrine
D Mesatonum
E Physostigmine
185 A 45-year-old patient suffers from neurosis characterized by
irritability, sleeplessness, motiveless anxiety. What drug would eliminate all the
symptoms?
A Levodopa
B Pyracetam
C Valerian extract
D Diazepam *
E Caffeine sodium benzoate
186 Acetylcholine IS NOT a specific neurotransmitter at:
A Sympathetic ganglia
B Sympathetic postganglionic nerve endings*
C Parasympathetic ganglia
D Parasympathetic postganglionic nerve endings
E None of the above
187 Muscarinic receptors are located in:
A Autonomic ganglia
B Skeletal muscle neuromuscular junctions
C Autonomic effector cells*
D Sensory carotid sinus baroreceptor zone
E All of the above
188 Indicate the location of M2 cholinoreceptor type:
A Heart*
B Glands
C Smooth muscle
D Endothelium
E All of the above
189 The symptoms of mushroom poisoning include all of the following
EXCEPT:
A Salivation, lacrimation, nausea, vomiting
B Dryness of mouth, hyperpyrexia, hallucination*
C Headache, abdominal colic
D Bradycardia, hypotension and shock
E None of the above
190 Which of the following cholinomimetics activates both muscarinic
and nicotinic receptors?
A Lobeline
B Pilocarpine
C Nicotine
D Carbacholine*
E Adrenalin hydrochloride
191 Indicate a cholinomimetic agent, which is related to direct-acting
drugs:
A Aceclidine
B Cytitone
C Carbacholine*
D Proserine
E Dipiroxim
192 Characteristics of carbacholine include all of the following EXCEPT:
A It decreases intraocular pressure
B It causes mydriasis*
C It exerts both nicotinic and muscarinic effects
D It is resistant to acethylcholiesterase
E All of the above
193 Acetylcholine IS NOT used in clinical practice because:
A It is very toxic
B The doses required are very high
C It is very rapidly hydrolyzed*
D It is very costly
E None of the above
194 Local anesthetics produce:
A Analgesia, amnesia,
B Blocking pain sensation without loss of consciousness *
C Alleviation of anxiety and pain with an altered level of consciousness
D A stupor or somnolent state
E Loss of consciousness
195 A good local anesthetic agent should cause:
A Local irritation and tissue damage
B Systemic toxicity
C Fast onset and long duration of action*
D Vasodilatation
E Heart failure
196 Which one of the following groups is responsible for the duration of
the local anesthetic action?
A Intermediate chain*
B Lipophylic group
C Ionizable group
D Amino group
E All of the above
197 Indicate the local anesthetic agent, which has a shorter duration of
action:
A Lidocaine
B Procaine*
C Bupivacaine
D Ropivacaine
E Ultracaine
198 Which one of the following groups is responsible for the potency and
the toxicity of local anesthetics?
A Ionizable group
B Intermediate chain
C Lipophylic group*
D Amino-group
E All of the above
199 Indicate the drug, which has greater potency of the local anesthetic
action:
A Lidocaine
B Bupivacaine*
C Procaine
D Mepivacaine
E Anesthesine
200 Ionizable group is responsible for:
A The potency
B The duration of action
C The ability to diffuse to the site of action*
D Toxicity
E All of the above
201 Which one of the following local anesthetics is an ester of benzoic
acid?
A Lidocaine
B Procaine
C Ropivacaine
D Ultracaine
E Cocaine*
202 Indicate the local anesthetic, which is an ester of paraaminobenzoic
acid:
A Mepivacaine
B Cocaine
C Procaine*
D Lidocaine
E Ultracaine
203 Which of the following local anesthetics is an acetanilide derivative?
A Tetracaine
B Lidocaine*
C Cocaine
D Procaine
E Anesthesine
204 Indicate the local anesthetic, which is a toluidine derivative:
A Lidocaine
B Bupivacaine
C Prilocaine*
D Procaine
E cocaine
205 Which of the following local anesthetics is a thiophene derivative?
A Procaine
B Ultracaine*
C Lidocaine
D Mepivacaine
E Bupivacaine
206 Local anesthetics are:
A Weak bases*
B Weak acids
C Salts, which formed by weak acid and strong base
D Salts, which formed by strong acid and weak base
E None of the above
207 For therapeutic application local anesthetics are usually made
available as salts for the reasons of:
A Less toxicity and higher potency
B Higher stability
C Greater lipid solubility
D Less local tissue damage and more potency
E More stability and greater water solubility*
208 Which of the following statements is not correct for local anesthetics?
A In a tissue they exist either as an uncharged base or as a cation
B A charged cationic form penetrates biologic membranes more readily
than an uncharged form*
C Local anesthetics are much less effective in inflamed tissues
D Low ph in inflamed tissues decreases the dissociation of nonionized
molecules
E Local anesthetics are weak bases
209 Which one of the following statements about the metabolism of local
anesthetics is incorrect?
A Metabolism of local anesthetics occurs at the site of administration*
B Metabolism occurs in the plasma or liver but not at the site of
administration
C Ester group of anesthetics like procaine, are metabolized systemically
by pseudocholinesterase
D Amides such as lidocaine, are metabolized in the liver by microsomal
mixed function oxidases
E In a tissue they exist either as an uncharged base or as a cation
210 Indicate the anesthetic agent of choice in patient with a liver disease:
A Lidocaine
B Bupivacaine
C Procaine*
D Etidocaine
E Mepivacaine
211 Which of the following local anesthetics is preferable in patient with
pseudocholinesterase deficiency?
A Procaine
B Ropivacaine*
C Tetracaine
D Benzocaine
E Cocaine
212 The primary mechanism of action of local anesthetics is:
A Activation of ligand-gated potassium channels
B Blockade of voltage-gated sodium channels*
C Stimulation of voltage-gated N-type calcium channels
D Blockade the GABA-gated chloride channels
E Stimulation of voltage-gated sodium channels
213 Which of the following local anesthetics is more water-soluble?
A Tetracaine
B Etidocaine
C Procaine*
D Bupivacaine
E Ultracaine
214 Indicate the local anesthetic, which is more lipid-soluble:
A Bupivacaine*
B Lidocaine
C Mepivacaine
D Procaine
E Ultracaine
215 The more lipophylic drugs:
A Are more potent
B Have longer duration of action
C Bind more extensively to proteins
D Better penetration
E All of the above*
216 Indicate the function, which the last to be blocked:
A Pain
B Temperature
C Muscle spindles
D Motor function*
E Touch
217 Which of the following local anesthetics is an useful antiarrhythmic
agent?
A Cocaine
B Lidocaine*
C Bupivacaine
D Ropivacaine
E Procaine
218 Indicate the route of local anesthetic administration, which is
associated with instillation within epidural or subarachnoidal spaces:
A Topical anesthesia
B Infiltrative anesthesia
C Regional anesthesia
D conductive anesthesia
E Spinal anesthesia*
219 The choice of a local anesthetic for specific procedures is usually
based on:
A The duration of action
B Water solubility
C Capability of rapid penetration through the skin or mucosa with
limited tendency to diffuse away from the site of application
D Absents of the allergic reaction
E All of the above*
220 Which of the following local anesthetics is a short-acting drug?
A Procaine*
B Tetracaine
C Bupivacaine
D Cocaine
E Ropivacaine
221 Indicate the local anesthetic, which is a long-acting agent:
A Lidocaine
B Bupivacaine*
C Procaine
D Mepivacaine
E Ultracaine
222 The anesthetic effect of the agents of short and intermediate duration
of action can not be prolonged by adding:
A Epinephrine
B Norepinephrine
C Dopamine*
D Phenylephrine
E Ephedrine
223 A vasoconstrictor does not:
A Retard the removal of drug from the injection site
B Hence the chance of toxicity
C Decrease the blood level
D Reduce a local anesthetic uptake by the nerve*
E Increase the duration of local anesthesia
224 Vasoconstrictors ar e less effective in prolonging anesthetic properties
of:
A Procaine
B Bupivacaine*
C Lidocaine
D Mepivacaine
E Ultracaine
225 Which of the following local anesthetics is only used for surface or
topical anesthesia?
A Cocaine*
B Tetracaine
C Procaine
D Bupivacaine
E Lidocaine
226 Indicate the local anesthetic, which is mainly used for regional nerve
block anesthesia:
A Dibucaine
B Bupivacaine*
C Tetracaine
D Cocaine
E Anesthesine
227 Which of the following local anesthetics is used for infiltrative and
regional anesthesia?
A Procaine
B Lidocaine
C Ultracaine
D Mepivacaine
E All of the above*
228 Indicate the local anesthetic, which is used for spinal anesthesia:
A Tetracaine*
B Cocaine
C Dibucaine
D Bupivacaine
E Anesthesine
229 Which of the following local anesthetics is called a universal
anesthetic?
A Procaine
B Ropivacaine
C Lidocaine*
D Bupivacaine
E Anesthesine
230 The most serious toxic reaction to local anesthetics is:
A Seizures
B Cardiovascular collapse
C Cardiac arrest
D Respiratory failure
E All of the above
231 Correct statements concerning cocaine include all of the following
EXCEPT:
A Cocaine is often used for nose and throat procedures
B Limited use because of abuse potential
C Myocardial depression and peripheral vasodilatation*
D Causes sympathetically mediated tachycardia and vasoconstriction
E Can case addiction
232 Which of the following local anesthetics is more cardiotoxic?
A Procaine
B Bupivacaine*
C Lidocaine
D Ultracaine
E Mepivacaine
233 Most local anesthetics can cause:
A Depression of abnormal cardiac pacemaker activity
B Depression of the strength of cardiac contraction
C Cardiovascular collapse
D Depression of the cardiac excitability and conduction
E All of the above*
234 Which one of the following local anesthetics causes
methemoglobinemia?
A Prilocaine*
B Procaine
C Lidocaine
D Ultracaine
E Ropivacaine
235 Procaine has all of the following properties EXCEPT:
A It has ester linkage
B Its metabolic product can inhibit the action of sulfonamides
C It readily penetrates the skin and mucosa*
D It is relatively short-acting
E It in derivative of the p-aminobenzoic acid
236 Correct statements concerning lidocaine include all of the following
EXCEPT:
A It is a universal anesthetic
B It has ester linkage*
C It widely used as an antiarrhythmic agent
D It is metabolized in liver
E It has amide group
237 Which of the following local anesthetics is more likely to cause
allergic reactions?
A Lidocaine
B Bupivacaine
C Procaine*
D Ropivacaine
E Ultracaine
238 Tetracaine has all of the following properties EXCEPT:
A Slow onset
B Low potency*
C Long duration
D High toxicity
E It has ester linkage
239 Correct statements concerning bupivacaine include all of the
following EXCEPT:
A It has low cardiotoxicity*
B It has amide linkage
C It is a long-acting drug
D An intravenous injection can lead to seizures
E It is cardiotoxic*
240 Sympathetic stimulation is mediated by:
A Release of norepinephrine from nerve terminals
B Activation of adrenoreceptors on postsynaptic sites
C Release of epinephrine from the adrenal medulla
D Dopamine
E All of the above*
241 Characteristics of epinephrine include all of the following EXCEPT:
A It is synthesized into the adrenal medulla
B It is synthesized into the nerve ending*
C It is transported in the blood to target tissues
D It directly interacts with and activates adrenoreceptors
E It stimulates glycogenolysis
242 Which of the following sympathomimetics acts indirectly?
A Epinephrine
B Norepinephrine
C Dopamine
D Ephedrine*
E Methoxamine
243 Catecholamine includes following EXCEPT:
A Ephedrine*
B Epinephrine
C Isoprenaline
D Norepinephrine
E Dopamine
244 Epinephrine decreases intracellular camp levels by acting on:
A α 1 receptor
B α 2 receptor*
C beta 1 receptor
D beta 2 receptor
E D-receptors
245 Which of the following statements is not correct?
A Alfa receptors increase arterial resistance
B Beta 2 receptor promote smooth muscle relaxation
C The skin and splanchic vessels have predominantly alfa receptors
D Vessels in a skeletal muscle may constrict or dilate depending on
whether alfa or beta 2 receptors are activated
E Skeletal muscle vessels have predominantly alfa receptors and
constrict in the presence of epinephrine and norepinephrine*
246 Direct effects on the heart are determined largely by:
A Alfa 1 receptor
B Alfa 2 receptor
C Beta 1 receptor*
D Beta 2 receptor
E dofamino-receptors
247 Which of the following effects is related to direct beta 1
-adrenoreceptor stimulation?
A Bronchodilation
B Vasodilatation
C Tachycardia*
D Bradycardia
E Decreasing of the heart conductance
248 Distribution of alfa adrenoreceptor subtypes is associated with all of
the following tissues except those of:
A Heart*
B Blood vessels
C Prostate
D Pupillary dilator muscle
E Bronchial glands
249 Beta adrenoreceptor subtypes are contained in all of the following
tissues EXCEPT:
A Bronchial muscles
B Heart
C Pupillary dilator muscle*
D Fat cells
E Bronchial glands
250 In which of the following tissues both alfa and beta 1 adrenergic
stimulation produces the same effect?
A Blood vessels
B Intestine*
C Uterus
D Bronchial muscles
E Heart
251 The effects of sympathomimetics on blood pressure are associated
with their effects on:
A The heart
B The peripheral resistance
C The venous return
D The blood vessels
E All of the above*
252 A relatively pure alfa agonist causes all of the following effects
EXCEPT:
A Increase peripheral arterial resistance
B Increase venous return
C Has no effect on blood vessels*
D Reflex bradycardia
E Muscle contraction
253 A nonselective beta receptor agonist causes all of the following effects
EXCEPT:
A Increase cardiac output
B Increase peripheral arterial resistance*
C Decrease peripheral arterial resistance
D Decrease the mean pressure
E Decrease need of miocardium in oxygen
254 Which of the following statement is not correct?
A Αlfa agonists cause miosis*
B Αlfa agonists cause mydriasis
C Beta antagonists decrease the production of aqueous humor
D Αlfa agonists increase the outflow of aqueous humor from the eye
E Alfa antagonists case expansion of peripheral blood vessels
255 A bronchial smooth muscle contains:
A Αlfa 1 receptor
B Αlfa 2 receptor
C Beta 1 receptor
D Dofamino-receptor
E Beta 2 receptor*
256 All of the following agents are beta receptor agonists EXCEPT:
A Epinephrine
B Isoproterenol
C Methoxamine*
D Dobutamine
E Norepinephrine
257 Which of the following drugs causes bronchodilation without
significant cardiac stimulation?
A Isoprenaline
B Orciprenaline sulfate
C Terbutaline*
D Xylometazoline
E Methoxamine
258 Αlfa-receptor stimulation includes all of the following effects
EXCEPT:
A Relaxation of gastrointestinal smooth muscle
B Contraction of bladder base
C Contraction of uterus and prostate
D Stimulation of insulin secretion*
E Stimulation of platelet aggregation
259 Beta 1 receptor stimulation includes all of the following effects
EXCEPT:
A Increase in contractility
B Bronchodilation*
C Tachycardia
D Increasing strength and heart rate
E Increase in conduction velocity in the atrioventricular node
260 Beta 2 receptor stimulation includes all of the following effects
EXCEPT:
A Stimulation of renin secretion
B Fall of potassium concentration in plasma
C Relaxation of uterus
D Relaxation of bladder
E Tachycardia*
261 Hyperglycemia induced by epinephrine is due to:
A Gluconeogenesis (beta 2 )
B Inhibition of insulin secretion (alfa)
C Stimulation of glycogenolysis (beta 2 )
D All of the above*
E None of above
262 Which of the following effects is associated with beta 3 -receptor
stimulation?
A Lipolysis*
B Decrease in platelet aggregation
C Bronchodilation
D Tachycardia
E Bradicardia
263 Which of the following statements is not correct?
A Epinephrine acts on both alfa- and beta-receptors
B Norepinephrine has a predominantly beta action*
C Methoxamine has a predominantly alfa action
D Norepinephrine acts on both alfa- and beta-receptors
E Isoprenaline has a predominantly beta action
264 Indicate the drug, which is a direct-acting both alfa- and beta-receptor
agonist:
A Norepinephrine*
B Methoxamine
C Isoproterenol
D Ephedrine
E Metoprolol
265 Which of the following agents is an alfa-1, alfa-2, beta-1, beta-2
receptor agonist?
A Methoxamine
B Albuterol
C Epinephrine*
D Metoprolol
E Norepinephrine
266 Indicate the direct-acting sympathomimetic, which is an alfa-1, alfa-2,
beta-1 receptor agonist:
A Isoproterenol
B Ephedrine
C Dobutamine
D Metoprolol
E Norepinephrine*
267 Which of the following agents is an alfa-1 -selective agonist?
A Norepinephrine
B Methoxamine*
C Ritodrine
D Ephedrine
E Isoproterenol
268 Indicate the alfa 2 -selective agonist:
A Xylometazoline*
B Epinephrine
C Norepinephrine
D Dobutamine
E Methoxamine
269 Which of the following agents is a nonselective beta receptor agonist?
A Norepinephrine
B Epinephrine
C Ephedrine
D Isoproterenol*
E Dobutamine
270 Indicate the beta 1 -selective agonist:
A Isoproterenol
B Dobutamine*
C Metaproterenol
D Epinephrine
E Norepinephrine
271 Which of the following sympathomimetics is a beta 2 -selective
agonist?
A Terbutaline*
B Xylometazoline
C Ephedrine
D Isoproterenol
E Dobutamine
272 Indicate the indirect-acting sympathomimetic agent:
A Epinephrine
B Norepinephrine
C Phenylephrine
D Ephedrine*
E Isoproterenol
273 Epinephrine produces all of the following effects except:
A Positive inotropic action on the heart (beta 1 receptor)
B Increase peripheral resistance (alfa receptor)
C Predominance of alfa effects at low concentration*
D Positive chronotropic action on the heart (beta 1 receptor)
E Skeletal muscle blood vessel dilatation (beta 2 receptor)
274 Epinephrine produces all of the following effects except:
A Decrease in oxygen consumption*
B Increase in oxygen consumption
C Bronchodilation
D Hyperglycemia
E Mydriasis
275 Epinephrine is used in the treatment of all of the following disorders
except:
A Bronchospasm
B Anaphylactic shock
C Cardiac arrhythmias*
D Open-angle glaucoma
E Hypoglycemic coma
276 Compared with epinephrine, norepinephrine produces all of the
following effects except:
A Similar effects on beta 1 receptors in the heart and similar potency at
an alfa receptor
B Decrease the mean pressure below normal before returning to the
control value*
C Significant tissue necrosis if injected subcutaneously
D Increase systolic blood pressure
E Increase diastolic blood pressure
277 Norepinephrine produces:
A Vasoconstriction*
B Vasodilatation
C Bronchodilation
D Decreased potassium concentration in the plasma
E Increased blood pressure
278 Which of the following direct-acting drugs is a relatively pure alfa
agonist, an effective mydriatic and decongestant and can be used to raise blood
pressure?
A Epinephrine
B Norepinephrine
C Phenylephrine*
D Ephedrine
E Reserpine
279 Characteristics of methoxamine include all of the following except:
A It is a direct-acting alfa 1 -receptor agonist
B It increases heart rate, contractility and cardiac output*
C It causes reflex bradycardia
D It increases total peripheral resistance
E Induced vasoconstriction
280 Which of the following agents is an alfa 2 -selective agonist with
ability to promote constriction of the nasal mucosa?
A Xylometazoline*
B Phenylephrine
C Methoxamine
D Epinephrine
E Norepinephrine
281 Indicate the sympathomimetic, which may cause hypotension,
presumably because of a clonidine-like effect:
A Methoxamine
B Phenylephrine
C Xylometazoline*
D Isoproterenol
E Epinephrine
282 Isoproterenol is:
A Both an alfa- and beta-receptor agonist
B Beta 1 -selective agonist
C Alfa receptor agonist
D Beta 2 -selective agonist
E Nonselective beta receptor agonist*
283 Isoproterenol produces all of the following effects except:
A Increase in cardiac output
B Fall in the diastolic pressure
C Fall in the mean arterial pressure
D Bronchoconstriction*
E Tachycardia
284 Characteristics of dobutamine include all of the following except:
A It is a relatively beta 1 -selective synthetic catecholamine
B It is used to treat bronchospasm*
C It increases atrioventricular conduction
D In over dosage can provoke arrhytmia
E It causes minimal changes in heart rate and systolic pressure
285 Characteristics of salmeterol include all of the following except:
A It is a potent selective beta 2 agonist
B It causes uterine relaxation
C It stimulates heart rate, contractility and cardiac output*
D It is used in the therapy of asthma
E A) and B)
286 Characteristics of ephedrine include all of the following except:
A It acts primarily through the release of stored cathecholamines
B It is a mild CNS stimulant
C It causes tachyphylaxis with repeated administration
D It decreases arterial pressure*
E A) and C)
287 Ephedrine causes:
A Miosis
B Sedatation effect on CNS
C Bronchodilation*
D Hypotension
E Bradycardia
288 Compared with epinephrine, ephedrine produces all of the following
features except:
A It is a direct-acting sympathomimetic*
B It has oral activity
C It is resistant to MAO and has much longer duration of action
D Its effects are similar, but it is less potent
E B) and D)
289 Which of the following sympathomimetics is preferable for the
treatment of chronic orthostatic hypotension?
A Epinephrine
B Norepinephrine
C Ephedrine*
D Salbutamol
E Dobytamine
290 Indicate the sympathomimetic drug, which is used in a hypotensive
emergency:
A Xylometazoline
B Ephedrine
C Terbutaline
D Dobutamine
E Phenylephrine*
291 Which of the following sympathomimetics is preferable for the
emergency therapy of cardiogenic shock?
A Epinephrine
B Dobutamine*
C Isoproterenol
D Methoxamine
E Salbutamol
292 Indicate the sympathomimetic agent, which is combined with a local
anesthetic to prolong the duration of infiltration nerve block:
A Epinephrine*
B Xylometazoline
C Salbutamol
D Isoproterenol
E Dobutamine
293 Which of the following sympathomimetics is related to short-acting
topical decongestant agents?
A Xylometazoline
B Terbutaline
C Phenylephrine*
D Norepinephrine
E Salbutamol
294 Indicate the long-acting topical decongestant agents:
A Epinephrine
B Norepinephrine
C Phenylephrine
D Xylometazoline*
E Salbutamol
295 Which of the following topical decongestant agents is an alfa 2
-selective agonist?
A Phenylephrine
B Xylometazoline*
C Ephedrine
D Epinephrine
E Norepinephrine
296 Indicate the sympathomimetic, which may be useful in the emergency
management of cardiac arrest:
A Methoxamine
B Phenylephrine
C Epinephrine*
D Xylometazoline
E Salbutamol
297 Which of the following sympathomimetics is used in the therapy of
bronchial asthma?
A Fenoterol*
B Norepinephrine
C Epinephrine
D Methoxamine
E Dobutamine
298 Indicate the agent of choice in the emergency therapy of anaphylactic
shock:
A Methoxamine
B Terbutaline
C Salbutamol
D cNorepinephrine
E Epinephrine*
299 Which of the following sympathomimetics is an effective mydriatic
agent?
A Salbutamol
B Phenylephrine*
C Dobutamine
D Norepinephrine
E Fenoterol
300 The adverse effects of sympathomimetics include all of the following
except:
A Drug-induced parkinsonism*
B Cerebral hemorrhage
C Pulmonary edema
D Myocardial infarction
E Ventricular arrhythmias
301 Which of the following drugs is a nonselective alfa receptor
antagonist?
A Prazosin
B Phentolamine*
C Metoprolol
D Reserpine
E Salbutamol
302 Indicate the alfa 1 -selective antagonist:
A Phentolamine
B Dihydroergotamine
C Prazosin*
D Labetalol
E Atenolol
303 Which of the following drugs is an nonselective beta receptor
antagonist?
A Metoprolol
B Atenolol
C Propranolol*
D Acebutolol
E Salbutamol
304 Indicate the beta 1 -selective antagonist:
A Propranolol
B Metoprolol*
C Carvedilol
D Sotalol
E Propranolol
305 Which of the following agents is a beta 2 –selective antagonist?
A Acebutolol
B Pindolol
C Labetolol
D Ergotamin
E Butoxamine*
306 Which of the following is not a side effect of the cholinoreceptor
blocker (Atropine)?
A Increased pulse
B Urinary retention *
C Constipation
D Mydriasis
E Photophobia
307 Which of the following is not a side effect of the Captopril?
A Rash
B Angioedema
C Cough
D Congestion*
E Head pain
308 Which of the following is not a side effect of the Clonidine?
A Hypertension
B Asthma*
C Dry oral cavity
D Lethargic behavior
E Nausea
309 Which of the following is not an effect of the drug (Isoflurane)?
A Elevated lipid levels*
B Nausea
C Increased blood flow to the brain.
D Decreased respiratory function
E Head pain
310 Which of the following is not an effect of the drug (Midazolam)?
A Amnesia
B Decreased respiratory function
C Anesthetic*
D Dizziness
E Head pain
311 Which of the following is not an effect of the drug (Clozapine)?
A Agranulocytosis
B Antipsychotic
C Used for Schizophrenia
D Increased appetite*
E Constipation
312 Which of the following is not treated with (Epinephrine)?
A Renal disease*
B Asthma
C Hypotension
D Glaucoma
E Shock
313 Which of the following is not treated with (Ephedrine)?
A COPD*
B Hypotension
C Congestion
D Incontinence
E Bronchospasm
314 Which of the following is not treated with Barbiturates?
A Seizures
B Hypotension*
C Insomnia
D Anxiety
E Epilepsy
315 Which of the following are not treated with opioid analgesics like
(dextromethorphan and methadone)?
A Pulmonary Edema
B Cough suppression
C Sedation*
D Pain
E Seizures
316 Pupil dilatation occurs with:
A neostigmine
B propranolol
C atenolol
D codeine *
E ganglion blocker
317 Pupil dilatation occurs with:
A neostigmine
B cocaine *
C atenolol
D metoprolol
E ganglion blocker
318 Procaine
A metabolized primarily in the liver
B seldom combined with a vasoconstrictor because it inhibits NE uptake
C interacts with Na+ channels predominantly in its cationic form *
D used more frequently to produce epidural anesthesia than is tetracaine
E reaches its site of action more rapidly in infected tissues compared to
normal tissue
319 Mepivacaine most closely resembles which of the following drugs?
A Lidocaine
B Anesthesine
C Tetracaine
D Chloroprocaine
E Ropivacaine *
320 Local anesthetic agents such as procaine:
A block nerve conduction in part by causing depolarization via
inhibition of K+ channels
B block conduction in motor fibers more rapidly than in pain fibers
C cause convulsions when excessive amounts reach the brain *
D cause local vasoconstriction, which prolongs their duration of action
E inhibit resting Na+ channels more readily than those activated by
depolarization
321 Which of the following drugs and doses would be expected to have
the quickest onset of sensory blockade during a brachial plexus nerve block?
A 20 ml of 2% 2-chloroprocaine
B 20 ml of 2% lidocaine. *
C 20 ml of 0.5% ropivacaine
D 20 ml of 0.5% bupivacaine
E 20 ml of 0.5% tetracaine
322 Bupivacaine:
A Contains amide linkage; administered intravenously for treatment of
cardiac arrhythmias
B Contains ester linkage; hydrochloride salt can be used topically to
anesthetize mucous membranes
C Contains ester linkage; shortest duration of all local anaesthetics used
clinically
D Contains amide linkage; duration of action is longer than that of
lidocaine *
E Only employed topically on the skin and mucous membranes
323 Lidocaine:
A Contains amide linkage; administered intravenously for treatment of
cardiac arrhythmias*
B Contains ester linkage; hydrochloride salt can be used topically to
anesthetize mucous membranes
C Contains ester linkage; shortest duration of all local anaesthetics used
clinically
D Contains amide linkage; duration of action is longer than that of
bupivacaine
E Only employed topically on the skin and mucous membranes
324 Procaine:
A Contains amide linkage; administered intravenously for treatment of
cardiac arrhythmias
B Contains ester linkage; hydrochloride salt can be used topically to
anesthetize mucous membranes
C Contains ester linkage; shortest duration of all local anaesthetics used
clinically*
D Contains amide linkage; duration of action is longer than that of
bupivacaine
E Only employed topically on the skin and mucous membranes
325 Tetracaine:
A Contains amide linkage; administered intravenously for treatment of
cardiac arrhythmias
B Contains ester linkage; hydrochloride salt can be used topically to
anesthetize mucous membranes
C Contains ester linkage; shortest duration of all local anaesthetics used
clinically
D Contains amide linkage; duration of action is longer than that of
bupivacaine
E Only employed topically on the skin and mucous membranes *
326 Which of the following is FALSE regarding procaine?
A metabolized in plasma
B use is confined with infiltration anesthesia and diagnostic nerve block
C short duration local anesthetic
D high potency *
E none of the above are false (all true)
327 Which of the following is TRUE regarding bupivacaine?
A more potent than procaine *
B an ester
C metabolized in plasma
D short duration of action
E contraindicated for use in epidural anesthesia
328 Which of the following statements is FALSE regarding the effects of
local anesthetics on sensory modalities and motor function?
A small fiber axons blocked more quickly than large fiber axons
B when a patient is waking up from anesthesia, their foot would regain
sensation before their knee
C faster firing axons are blocked first
D proprioceptive fibers are blocked more easily than pain fibers *
E analgesia precedes the loss of non-nociceptive sensations and motor
function. Recovery occurs in the opposite order.
329 Which of the following local anesthetics is useful for elderly patients
or those with cardiovascular disease?
A Bupivacaine
B Lidocaine
C mepivicaine *
D procaine
E tetracaine
330 Which of the following is FALSE regarding the structural features of
local anesthetics is (LA)?
A The hydrophobic aromatic moiety permits diffusion through the
membrane to the site of action, which is the intracellular side of the sodium
channel pore
B LA are weak bases
C LA have an ester or an amide linkage. Those with esters are
hydrolyzed in the plasma whereas amides are degraded in the liver
D The charged form of an LA (BH+) has the LEAST of the channel
blocking activity *
E The uncharged form of an LA (B) is lipid soluble and thus can diffuse
through lipid bilayers
331 Which of the following is TRUE regarding side effects of local
anesthetics is (LA)?
A amides are more likely to cause hypersensitivity reactions than esters
B low doses of LA can cause convulsions
C LA are often mixed with epinephrine when working on areas supplied
by a terminal artery such as the digits or the penis
D A concern when doing spinal anesthesia are indirect effects on the
cardiovascular system due to sympathetic blockade *
E High systemic concentrations of LA cannot cause conduction failure,
ventricular arrhythmias or fibrillation of the myocardium
332 Bupivacaine:
A An amide. Often used in continuous epidural anesthesia *
B An ester. 10 times more potent than procaine. Longer duration of
action
C An amide. 2-3 times more potent than procaine. One of the most
widely used LA
D An ester. First synthetic LA
E None of the above
333 Lidocaine:
A An amide. Often used in continuous epidural anesthesia
B An ester. 10 times more potent than procaine. Longer duration of
action
C An amide. 2-3 times more potent than procaine. One of the most
widely used LA*
D An ester. First synthetic LA
E None of the above
334 Procaine:
A An amide. Often used in continuous epidural anesthesia
B An ester. 10 times more potent than procaine. Longer duration of
action
C An amide. 2-3 times more potent than procaine. One of the most
widely used LA
D An ester. First synthetic LA*
E None of the above
335 Tetracaine:
A An amide. Often used in continuous epidural anesthesia
B An ester. 10 times more potent than procaine. Longer duration of
action*
C An amide. 2-3 times more potent than procaine. One of the most
widely used LA
D An ester. First synthetic LA
E None of the above
336 The rate of uptake of an anesthetic gas from the lung:
A is proportional to the solubility of the gas in blood. *
B increases as the partial pressure of the gas in tissues approaches that in
the alveolus
C is decreased when pulmonary ventilation is increased
D increases as the ratio of the partial pressures in the inspired and
expired gas approaches unity
E increases if MAC has been lowered by prior administration of
preanesthetic medication
337 Compared to desflurane at equal values of MAC, isoflurane:
A is more irritating to the airway
B will be administered at a greater concentration
C will induce anesthesia more slowly *
D is more likely to induce hypertension
E is more likely to induce tachycardia
338 All of the following statements about nitrous oxide are true EXCEPT:
A surgical anesthesia at normal atmospheric pressure requires the
concomittant use of other agents
B produces analgesia at subanesthetic partial pressures
C accumulates in fat, which slows emergence from prolonged surgical
anesthesia *
D its solubility in blood is low compared to other inhalational anesthetics
E produces little or no decrease in cardiac output or arterial pressure
339 Which of the following statements is MOST valid?
A The tension of isoflurane in blood is identical to its concentration if it
is the only anesthetic gas administered
B Low protein binding in blood explains the low solubility of
sevoflurane
C MAC is essentially an indicator of partial pressure
D Administered at equal MAC values at equilibrium, concentrations of
isoflurane and sevoflurane at their anesthetic site of action are similar. *
E Anesthetics act at the site of action in low micromolar concentrations
340 Where is a spinal anesthetic applied?
A Inside the membrane sheath surrounding the dorsal root ganglion
B In the subdural space
C In the subarachnoid space *
D In the grey matter of the spinal cord
E In the white matter of the spinal cord
341 Which of the following is not a symptom of shock?
A Cold clammy skin
B Hypotension
C Hypertension *
D Tachycardia
E Bronchospasm
342 What is an adverse reaction of an adrenergic drug depending on?
A Specific drug
B Drug dose
C Individual patient response
D The way of usage
E All of above*
343 Which drug has the greatest effect on the vascular system?
A Beta1-adrenergic blocking drug
B Alfa- adrenergic blocking drug*
C Antiadrenergic drugs
D Alfa-, beta-adrenergic blocking agents
E Beta2-adrenergic blocking drug
344 Which drug has the greatest effect on the adrenergic nerves in the
heart?
A Beta1-adrenergic blocking drug*
B Alfa- adrenergic blocking drug
C Antiadrenergic drugs
D Alfa-, beta-adrenergic blocking agents
E Beta2-adrenergic blocking drug
345 Which drug has the wider range of effects?
A Alfa-, beta-adrenergic blocking agents*
B Alfa1- adrenergic blocking drug
C Beta- adrenergic blocking drug
D Antiadrenergic drugs
E Alfa2- adrenergic blocking drug
346 What is the primary usage of beta-blockers agents?
A Hypotension
B Hypertension*
C Glaucoma
D Chest pain
E Myasthenia gravis
347 What is not considered for major use of cholinergic drugs?
A Glaucoma
B Hypertension*
C Myasthenia gravis
D Urinary retention
E Hypotension
348 What adverse reaction would be commonly noted with the usage of
cholinergic blockers?
A Blurred vision
B Dry mouth
C GI upset
D Expanding of the pupil
E All of above*
349 What drug is contraindicated for a patient taking adrenergic drugs for
high blood pressure?
A Bronchodilator
B Nasal decongestant*
C Aspirin
D Anesthetic
E Analgin
350 What is the neurotransmitter that inactivates acetylcholine?
A Acetyl cholinesterase*
B Lactase
C Alanine aminotranspherase
D Aspartate aminotranspherase
E Adenylatecyclase
351 Purpose of adrenergic drugs
A Shock*
B Hypotension
C Asthma
D Anesthesia
E Insomnia
352 Butyrophenone antipsychotic:
A Thioridazine
B Chlorpromazine
C Thiothixene
D Haloperidol *
E Fluphenazine
353 An example of an "atypical" antipsychotic agent:
A Thioridazine
B Thiothixene
C Risperidone *
D Chlorpromazine
E None of the above
354 Site(s) of chlorpromazine (Thorazine) receptor blockade:
A Alpha adrenergic receptor
B Muscarinic cholinergic receptor
C H1 histaminic receptor
D Serotonin - 5-HT2 receptor
E All of the above*
355 Antipsychotic drug with least extrapyramidal toxicity:
A Fluphenazine
B Haloperidol
C Clozapine *
D Thiothixene
E Chlorpromazine
356 Antipsychotic drug:least hypotensive effect:
A Chlorpromazine
B Thiothixene
C Clozapine
D Olanzapine *
E All about equal in hypotensive action
357 Antipsychotic endocrine effect(s):
A False -positive pregnancy tests
B Increased libido in women
C Decreased libido in men
D Amenorrhea -galactorrhea*
E All the above
358 Least likely to cause an increase in prolactin:
A Chlorpromazine
B Haloperidol
C Perphenazine
D Olanzapine *
E Thiothixene
359 Cardiovascular effects of "high-dose" (low-potency) phenothiazines:
A Hypertension
B Increased peripheral resistance
C Increased stroke volume
D Shortening of the Q-T interval
E None of the above*
360 Blockade of this receptor system most likely associated with Akathisia
side effect of antipsychotic treatment:
A Muscarinic cholinergic
B Alpha adrenergic
C Dopamine*
D serotonin
E GABA
361 Toxic-confusion states due to antipsychotic medication associated
with effects on this receptor system
A Alpha -adrenergic
B Dopamine
C Muscarinic cholinergic*
D Serotonergic
E None -of the above
362 Neurological adverse effect associated with antipsychotic drug use:
occurs late in therapy
A Parkinson's syndrome
B Akathisia
C Acute dystonic reactions
D Tardive dyskinesia*
E All of the above
363 Reasonable pharmacological step(s) to treat tardive dyskinesia:
A Increase antipsychotic drug dose
B Add a tricyclic antidepressants to the drug regimen
C Add benztropine (Cogentin)
D Start high-dose diazepam (30-40 milligrams per day)*
E No pharmacological intervention is helpful after tardive dyskinesia is
induced by antipsychotic treatment
364 Before the infiltration anesthesia was carried out, the test for
novocaine sensitivity was made to the patient. It was positive. Which of the
following drugs can be used for anesthesia in this case?
A * Lidocaine
B Streptomycin
C Cocaine
D Anaesthesinum
E Diсaine
365 In the psychiatric hospital, during the long-term treatment, symptoms
of the Parkinson's disease appeared in the patient’s condition. What drug was
administered to the patient?
A * Aminazine e
B Mezapam
C Sodium bromide
D Lithium Carbonate
E Nialamide
366 The unconscious patient was delivered to the admission department
of the hospital. The patient’s skin was cold, pupils of the eyes were narrowed,
breathing was difficult, blood pressure was decreased, the urinary bladder was
full. The diagnosis was morphine poisoning. What preparation should be used
as antagonist?
A * Naloxone
B Bemegride
C Cytiton
D Unithiol
E Sodium thiosulfate
367 The patient, who complained of dry mouth, photophobia, sight
disturbance, dry skin, tachycardia etc., was delivered to the admission
department. The administered diagnosis was belladonna alkaloids poisoning.
Which drugs should be used?
A * Neostigmine
B Diazepam
C Pilocarpine
D Armin
E Dipiroxim
368 The patient was diagnosed with acute myocardial infarction, which is
accompanied by the persistent pain behind the breastbone. The inefficiency of
previously administered drugs allowed the doctor to make neuroleptic
procedure. What neuroleptic drugs are used for this type of anesthesia?
A * Droperidol
B Metaperazyn
C Haloperidol
D Reserpine
E Aminazine e
369 The condition of patient with hypotension was gradually worsening.
The patient complained of dizziness, weakness, and he took some
nonprescription ephedrine drugs tablets. However, some improvement was
observed only after the first use of the pill. What phenomenon is caused by this
action?
A * Tachyphylaxis
B Sensitization
C Idiosyncrasy
D Habituation
E Cumulation
370 During the next attack of asthma, the patient inhaled medication,
known as bronchodilator or as the drug used in obstetric practice to prevent
miscarriages, with the help of the inhaler. What medical preparation was used
for the patient?
A * Fenoterol
B Salbutamol
C Adrenalini hydrochloridum
D Ephedrinum
E Theophylline
371 The man in age of 36 with traumatic brain injury complained of
weakened breathing, threadlike pulse, absent reflexes. What is the most
appropriate route of of piracetam administration in this case?
A * Intravenously
B Subcutaneously.
C Orally.
D Rectally.
E Inhalation
372 The woman in age of 35 complained of the pain associated with
delayed first childbirth period. What is the best drug to decrease the pain?
A * Promedol
B Analgin
C Morphine hydrochloridum.
D Codeine phosphas.
E Paracetamol
373 The patient complained of dizziness, thirst, swallowing difficulties,
poor sight was asked to the doctor. He had his rapid breathing, dilated pupils,
general arousal, loquacity and the blood pressure was 110 /70 mm Hg and the
pulse was 110 per minute. The following symptoms may indicate at the drug
overdose?
A * Atropine sulfas
B Morphine hydrochloridum
C Ephedrinum
D Chlorpromazine
E Caffeine
374 At the molecular level, best defined lithium effect:
A Effect on transmitters
B Effect on inositol phosphates*
C Effect on neurotransmitter receptors
D Effect on transmitters; Effect on inositol phosphates
E Effect on transmitters; Effect on inositol phosphates; Effect on
neurotransmitter receptors
375 Lithium carbonate is the preferred treatment for:
A Typical rendogenous depression
B Schizophrenia
C Anxiety
D Bipolar disorder*
E Enuresis
376 Lithium: polydipsia/polyuria due to:
A Excessive renal blood flow
B Collecting tubule in adequately responsive to ADH*
C Decreased GFR
D Reaction of concurrent amiloride administration
E None of the aboveї
377 Sympathetic nervous system anatomy and properties:
A Gangionic neurotransmitter: acetylcholine
B Generalized response upond sympathetic stimulation
C Thoraco -lumbar origin for preganglionic cell bodies
D All of the above*
E Gangionic neurotransmitter: acetylcholine; Generalized response
upond sympathetic stimulation
378 Comparing autonomic and somatic nerves:
A Somatic nerves, like autonomic nerves contain ganglia.
B Most of the fibers in the vagus nerve ar sensory*
C Denervated smooth muscle show spontaneous activity
D Moto r nerves are typically unmyelinated.
E Somatic nerves
379 Major anatomical site for integration of autonomic information:
A Cerebellum
B Sensory cortex
C Hypothalamus*
D Spinal cord--dorsal horn
E Cerebellum ; Sensory cortex
380 Division of the autonomic nervous system associated with diffuse
autonomic responses.
A Sympathetic*
B Parasympathetic
C Both
D Neither
E Sympathetic ; Parasympathetic
381 Activation of the sympathetic nervous system will caus which change
in the skeletal muscle versus cutaneous vascular beds.
A Vasoconstriction , vasoconstriction
B Vasodilatation , vasodilatation
C Vasodilatation , vasoconstriction*
D Vasoconstriction , vasodilation
E All of the above
382 Adrenergic receptor type(s) mediating pupillary dilation
A Beta -2
B Alpha -1*
C Muscarinic
D Serotonergic
E All of the above
383 Cholinergic receptor type that mediates vasodilation following low-
dose i.v. acetylcholine administration:
A Nicotinic
B Muscarinic*
C Nitric oxide receptor
D Substance P receptor
E All of the above
384 "True" acetylcholinesterase is found in:
A Glia
B Liver
C Erythrocytes*
D Plasma
E Glia ; Liver
385 Catalyzes rate-limiting step in catecholamine biosynthesis:
A DOPA decarboxylase
B Phenylethanolamine N-methyl transferase
C Tyrosine hydroxylase*
D Dopamine -beta-hydroxylase
E DOPA decarboxylase; Phenylethanolamine N-methyl transferase
386 Inhibited by drugs such as phenelzine or tranylcypromine;
A COMT (catechol-O-methyl transferase)
B MAO (monoamine oxidase)*
C Choline acetyltransferase
D Reuptake -I inhibitor
E All of the above
387 Concentation increased by epinephrine:
A Blood free fatty acids
B Blood glucose
C Skeletal muscle glycogen
D Blood free fatty acids; Blood glucose*
E All of the above
388 Due to receptor specificity, catecholamine LEAST likely to produce
bronchiolar smooth muscle relaxation:
A Epinephrine
B Terbutaline
C Phenylephrine*
D Phentolamine
E Epinephrine; Terbutaline
389 Alpha adrenergic receptor blocker
A Phentolamine
B Phenoxybenzamine
C Terbutaline
D Phentolamine; Phenoxybenzamine*
E All of the above
390 Alpha-adenergic receptor agonist:
A Terbutaline
B Atropine sulfas
C Methoxamine *
D Isoproterenol
E Terbutaline; Atropine sulfas
391 Parasympathetic direct cardiac effects:
A Decrease heart rate; increase contractility
B Increase heart rate; decrease contractility
C Decrease heart rate; decrease contractility *
D Increase AV nodal conduction velocity
E Decrease heart rate; increase contractility; Increase heart rate;
decrease contractility
392 Choline ester substrate for acetylcholinesterase:
A Carbachol
B Methacholine (Provocholine)*
C Both
D Neither
E All of the above
393 Alkaloid agonist acting at muscarinic, cholinergic receptors:
A DFP
B Pilocarpine *
C Physostigmine bromid
D Ipratropium
E All of the above
394 Effective in treating both organophosphate and muscarine
intoxication:
A Nicotine
B Echothiophate
C Atropine sulfas *
D Pilocarpine
E Nicotine; Echothiophate
395 Cholinergic activity on stomach acid secretion
A Increased *
B Decreased
C No changed
D One of the others is right
E Increased; Decreased
396 Most likely to reduce blood pressure by directly decreasing heart rate:
A Phentolamine
B Propranolol *
C Nitroprusside sodium
D Phenylephrine
E All of the above
397 From the point of view of Starling's law which antihypertensive would
be most likely to reduce contractility.
A Methoxamine
B Nitroprusside sodium*
C Propranolol
D Metoprolol
E All of the above
398 Negative inotropism
A Isoproterenol
B Epinephrine
C Diltiazem *
D Norepinephrine
E All of the above
399 Increases pulmonary congestion in congestive heart failure (CHF)
A Dopamine
B Metoprolol (Lopressor)*
C Nitroprusside sodium (Nipride)
D Digoxin (Lanoxin, Lanoxicaps)
E All of the above
400 Major neurotransmitter released at end organ effectors of the
thoracolumbar division of the autonomic nervous system:
A Dopamine
B Epinephrine
C Norepinephrine*
D Acetylcholine
E Dopamine; Epinephrine
401 Neurotransmitter of preganglionic fibers:
A Norepinephrine
B Substance P
C Epinephrine
D Acetylcholine*
E Norepinephrine; Substance P
402 "Fight or flight" activation of the ANS:
A Pupillary constriction
B Blood flow shifted from cutaneous beds to skeletal muscle*
C Blood glucose falls
D Bronchiolar constriction
E All of the above
403 Methoxamine (Vasoxyl)-induced bradycardia would be prevented by:
A Phentolamine (Regitine)
B Mecamylamine (Inversine)
C Atropine sulfas
D All of the above*
E Phentolamine (Regitine); Mecamylamine (Inversine)
404 Dopamine beta hydroxylase catalyzes:
A Tyrosine to DOPA
B DOPA to dopamine
C Dopamine to norepinephrine*
D Norepinephrine to epinephrine
E Tyrosine to DOPA; DOPA to dopamine
405 Primary mechaism for termination of norepinephrine and epinephrine
action:
A Metabolic transformation catalyzed by MAO
B Metabolic transformation catalyzed by COMT
C Diffusion away from the synaptic cleft and uptake at extraneuronal
sites
D Reuptake into nerve terminals*
E Metabolic transformation catalyzed by MAO; Metabolic
transformation catalyzed by COMT
406 Most potent at beta adrenergic receptors
A Epinephrine
B Isoproterenol *
C Norepinephrine
D Dopamine
E All of the above
407 alpha-2 receptor agonist; peripheral sympathomimetic
A Yohimbin
B Dobutamine
C Clonidine *
D Phenylephrine
E All of the above
408 Primary antihypertensive effect due to nitric oxide mediation of
smooth muscle relaxation.
A Atropine sulfas
B Nitroprusside sodium*
C Mecamylamine
D Captopril
E All of the above
409 Inhibits neurotransmitter enzymic degradation:
A Tubocurarine
B Phenoxybenzamine
C Physostigmine *
D Bretylium
E Tubocurarine; Phenoxybenzamine
410 Cardiac effects not like to be directly affected by the presence of an
anticholinesterase:
A Acetylcholine
B Methacholine
C Vagal stimulation
D Carbamylcholine *
E Acetylcholine; Methacholine
411 Indicate Pilocarpine action:
A Dry mouth
B Pupillary dilation
C Increased gastrointestinal tone*
D Bronchiolar relaxation
E All of the above
412 Preganglionic fibers terminating on adrenal medullary chromaffin
cells release:
A Norepinephrine
B Epinephrine
C Acetylcholine*
D Dopamine
E Substance P
413 Primary receptor type at autonomic ganglia:
A Adrenergic : beta 1
B Adrenergic : beta 2
C Cholinergic : muscarinic
D Cholinergic : nicotinic*
E Dopaminergic : D. 1
414 Vesicular protein important in docking with the presynaptic
membrane:
A Neurexin
B Syntaxin
C Saxitonin
D Synaptobrevin*
E All of the above
415 Rate-limiting step in acetylcholine synthesis:
A Choline acetyltransferase activity
B Vesicular protein synthesis
C Choline uptake*
D Acetylcholinesterase activity
E Availability of acetate
416 Influx of this ion promotes fusion between axoplasmic membrane and
nearby vesicles.
A Sodium
B Potassium*
C Calcium
D Chloride
E All of the above
417 Clostridium toxins:
A Inhibit acetylcholinesterase
B Prevent reuptake of choline
C Inhibit vesicular acetylcholine release*
D Prevent calcium influx
E All of the above
418 Denervation supersensitivity in skeletal muscle is mainly due to:
A Increase in receptor affinity
B No increase in the receptor number
C Proliferation of receptors*
D Increase in G-protein coupling efficiency
E Increase in acetylcholine release
419 Cholinergic receptor type that mediates the decrease in heart rate by
activating potassium channels:
A M1-- muscarinic
B M2-muscarinic*
C M3-muscarinic
D Nicotinic
E All of the above
420 Essential cofactor for the enzyme dopamine beta-hydroxylase:
A Pyridoxyl phosphate
B Ascorbate*
C Tetrahydrobiopterin
D Glycine
E All of the above
421 Phosphorylation of this enzyme is most likely to have an effect on
catecholamine biosynthesis:
A Phenylethanolamine N-methyltransferase
B Dopamine beta-hydroxylase
C Tyrosine hydroxylase*
D Dopa decarboxylase
E All of the above
422 Drugs activating this receptor are used in treating asthma:
A Beta 1 adrenergic
B Muscarinic cholinergic
C Beta 2 adrenergic*
D Nicotinic cholinergic
E All of the above
423 Epinephrine effects on the heart
A Increased rate
B Decreased contractility
C Coronary vasodilation
D Increased rate; Coronary vasodilation*
E All of the above
424 Receptor activation mainly responsible for positive inotropism:
A Alpha 1
B Beta 1*
C Dopamine D1
D Muscarinic cholinergic
E All of the above
425 Activates alpha receptors
A Isoproterenol
B Propranolol
C Phenylephrine *
D Terbutaline
E All of the above
426 Blocks cardiac isoproterenol effects
A Terbutaline
B Esmolol *
C Atropine sullfas
D Mecamylamin
E All of the above
427 Alpha agonist: vasoconstriction and elevates blood pressure:
A Metoprolol
B Methoxamine *
C Terbutaline
D Ipratropium
E All of the above
428 Nerve terminal reuptake inhibitor
A Methoxamine
B Cocaine*
C Reserpine
D Timolol
E All of the above
429 Alpha adrenoceptor COVALENT blocker:
A Propranolol
B Phenoxybenzamine *
C Phentolamine
D Pilocarpine
E All of the above
430 Orthostatic (postural) hypotension
A Beta receptor activation
B Alpha receptor activation
C Alpha receptor blocker*
D Dopamine receptor blockade
E All of the above
431 Norepinephrine pressor response blocked by:
A Mecamylamine
B Prazosin *
C Atropine sulfas
D Propranolol
E All of the above
432 Bronchodilation action belong to
A Ipratropium
B Timolol
C Albuterol
D Ipratropium ; Albuterol*
E All of the above
433 Positive chronotropic effects of epinephrine:
A Increased SA nodal potassium current
B Beta 1 receptor activation
C Mediated by G protein
D Beta 1 receptor activation; Mediated by G protein*
E All of the above
434 Maximal ß-adrenergic receptor desensitization depends on:
A Receptor occupancy by agonists
B An arrestin protein
C Receptor phosphorylation
D Receptor occupancy by agonists; An arrestin protein; Receptor
phosphorylation*
E All of the above
435 Most likely to increase myocardial afterload
A Angiotensin converting enzyme inhibitor (decreases angiotensin II
concentration)
B Propranolol
C Phenylephrine *
D Low -dose epinephrine
E All of the above
436 Pressor effects of epinephrine are blocked by this drug ("epinephrine
reversal")
A Propranolol
B Phentolamine *
C Phenylephrine
D Metoprolol
E All of the above
437 Decreases blood pressure
A Propranolol
B Mecamylamine
C Phentolamine
D All of the above*
E Paracetamol
438 Specific alpha2 receptor agonist
A Phenoxybenzamine
B Propranolol
C Guanfacine *
D Methoxamine
E All of the above
439 Centrally-acting antihypertensive drug
A Nitroprusside sodium
B Clonidine *
C Methoxamine
D Captopril
E All of the above
440 Isoproterenol (Isuprel): cardiopulmonary effects:
A Increases peripheral resistance; Positive inotropism*
B Increases peripheral resistance
C Positive inotropism
D Positive chronotropism
E None of the above
441 Drug causes pupillar dilation with no effect on accommodation:
A Pilocarpine
B Phenoxybenzamine
C Phentolamine
D Atropine sulfas
E Neostigmine *
442 Prevents blood pressure reduction seen with isoproterenol (Isuprel):
A Propranolol *
B Phenylephrine
C Atropine sulfas
D Phentolamine
E Esmolol
443 Beta-2 selective agonist:
A Metaproterenol *
B Phenylephrine
C Epinephrine
D Labetalol
E Phentolamine
444 Physiological effects associated with isoproterenol (Isuprel):
A Increased GI motility
B Increased peripheral resistance
C Bronchoconstriction
D Decreased heart rate
E Increased level of blood glucose*
445 Indicate action of salbutamolum:
A Bronchodilation*
B By I.V. injection only
C Decreases myocardial contractility
D Alpha adrenoceptor antagonist
E Significant heart rate increase
446 The magnitude of the cardiovascular response to norepinephrine is
increased by cocaine because:
A Cocaine decreases cholinergic receptor number
B Cocaine inhibits norepinephrine reuptake*
C Cocaine decreases N.E. metabolism by MAO
D Cocaine increases N.E. receptor number
E Cocaine increases conversion of norepinephrine to phenylephrine
447 Increases both magnitude of the blood pressure increase due to
phenylephrine and the heart rate decrease due to methacholine:
A Timolol
B Mecamylamine *
C Nitroprusside sodium
D Atropine sulfas
E Clonidine
448 Immediate biosynthetic precursor of epinephrine:
A L-DOPA
B Isoproterenol
C Doapmine
D Metaraminol
E Norepinephrine*
449 Therapeutic use of metaprolol:
A Renal vasodilator
B Positive inotrope in CHF
C Antihypertensive
D Antiarrhythmic drug*
E Bronchial asthma
450 Probably the neurotransmitter in sensory afferents
A Acetylcholine
B Bradykinin
C Substance P*
D Glycine
E Norepinephrine
451 Preganglionic fibers terminating on adrenal medullary chromaffin
cells release:
A Norepinephrine
B Epinephrine
C Acetylcholine*
D Dopamine
E Substance P
452 Primary receptor type at autonomic ganglia:
A Adrenergic : beta 1
B Adrenergic : beta 2
C Cholinergic : muscarinic
D Cholinergic : nicotinic*
E Dopaminergic : D1
453 Vesicular protein important in docking with the presynaptic
membrane:
A Neurexin
B Syntaxin
C Saxitonin
D Synaptobrevin*
E All of the above
454 Rate-limiting step in acetylcholine synthesis:
A Choline acetyltransferase activity
B Vesicular protein synthesis
C Choline uptake*
D Acetylcholinesterase activity
E Availability of acetate
455 Influx of this ion promotes fusion between axoplasmic membrane and
nearby vesicles.
A Sodium
B Potassium
C Calcium*
D Chloride
E All of the above
456 Clostridium toxins:
A Inhibit acetylcholinesterase
B Prevent reuptake of choline
C Inhibit vesicular acetylcholine release*
D Prevent calcium influx
E All of the above
457 Essential cofactor for the enzyme dopamine beta-hydroxylase:
A Pyridoxyl phosphate
B Ascorbate*
C Tetrahydrobiopterin
D Glycine
E All of the above
458 Phosphorylation of this enzyme is most likely to have an effect on
catecholamine biosynthesis:
A Phenylethanolamine N-methyltransferase
B Dopamine beta-hydroxylase
C Tyrosine hydroxylase*
D Dopa decarboxylase
E All of the above
459 Effects of Epinephrine:
A Limited effect on alpha receptors
B Increases heart rate, contributing to increase blood pressure*
C Epinephrine often reduces peripheral vascular resistance, especially at
high concentration
D Epinephrine tends to exhibit negative inotropic effects
E All of the above
460 Vasoconstrictive effects of epinephrine:
A Alpha -1 adrenergic receptor-mediated affecting precapillary
resistance vessels of the skin, kidney, and mucosa
B Veins
C Both*
D Neither
E All of the above
461 Renal effects relatively low epinephrine dose.
A Limited effect
B Beta 1 adrenergic receptor activation decreases renin release
C Significant reduction in renal blood flow*
D Significant increase in renal blood flow; mechanism similar to that
exhibited by low-dose dopamine
E All of the above
462 Cardiac effects associated with epinephrine:
A Positive chronotropic
B Positive inotropic
C Increased cardiac output
D Increased oxygen consumption
E All of the above*
463 Significant respiratory tract effects of epinephrine:
A Beta -2 receptor mediated bronchoconstriction
B Alpha -1 receptor-mediated bronchodilation
C Beta -1 receptor-mediated bronchodilation
D Beta -2 receptor-mediated bronchodilation*
E All of the above
464 Examples of epinephrine metabolic effects
A Insulin secretion reduced by beta2 adrenergic receptor activation
B Glucagon secretion: diminished by beta adrenergic receptor activation
C Free fatty acids: increased*
D Minimal calorigenic effect
E Glycolysis inhibition
465 Toxicities/adverse reactions associated with sympathomimetics
A Angina
B Hypertension ; cerebral hemorrhage
C Cardiac arrhythmias
D Anxiety reactions
E All the above*
466 Drugs antagonize epinephrine pressor effects:
A Phentolamine *
B Terbutaline
C Dopamine
D Dobutamine
E Atropine sulfas
467 Ventricular effects associated with epinephrine administration:
A Increased automaticity
B Increased ectopic pacemaker activity
C Increased conduction philosophy
D Increased contractility
E All the above*
468 Primary neurotransmitter released by postganglionic neurons of the
autonomic sympathetic system:
A Epinephrine
B Dopamine
C Dobutamine
D Norepinephrine*
E Phenylephrine
469 Decreased heart rate following norepinephrine infusion is most likely
due to:
A Direct norepinephrine activation of muscarinic receptors at the SA
node
B Heart rate cannot decrease following norepinephrine infusion because
norepinephrine activates beta-1 adrenergic receptors
C Activation of the baroreceptor system causing a reflex-mediated
decrease in heart rate*
D Peripheral vasodilation
E None of the above
470 Vascular effects of norepinephrine:
A Significantly decreases glomerularl filtration rates
B Effective in treating variant (Prinzmetal's) angina
C Norepinephrine pressor effects blocked by prazosin (Minipress)*
D Increased blood flow to liver, kidney, and skeletal muscle
E All of the above
471 Immediate synthetic precursor of norepinephrine:
A Epinephrine
B Tyrosine
C Tyrosine hydroxylase
D Dopamine*
E Dopa
472 CNS neurotransmitter associated with the basal ganglia and motor
control:
A Dopamine
B Acetylcholine
C Both*
D Neither
E All of the above
473 Low doses, this precursor of norepinephrine causes renovascular
dilation:
A Epinephrine
B Dopa
C Dopamine *
D Dobutamine
E Nitroprusside sodium
474 Pharmacological action(s) of dopamine (Intropin):
A Positive inotropism
B Promotes myocardial norepinephrine release
C Increases glomerular filtration rates (low-dose)
D Vasoconstriction by alpha-1 receptor activation (high-dose)
E All the above*
475 Significant therapeutic use for dopamine:
A Management of sleep cycles
B Treatment of Raynaud's phenomenon
C Treatment of cardiogenic/hypovolemic shock*
D Management of tachyarrhythmias
E All of the above
476 This drug has limited action at alpha-adrenergic receptors
A Phenylephrine
B Methoxamine
C Norepinephrine
D Isoproterenol *
E Prazosin
477 Effect of IV isoproterenol (Isuprel) infusions on blood pressure:
A Significant vasopressor effect
B Significant hypotensive effect
C Slight decrease in mean pressure with a significant decrease in
diastolic pressure*
D Significant increase in systolic pressure with minimal effect on
diastolic pressure
E All of the above
478 Adverse effects associated with isoproterenol (Isuprel) administration:
A Palpitations
B Tachycardia
C Arrhythmias
D Palpitations; Arrhythmias
E Palpitations; Arrhythmias; Tachycardia*
479 Simultaneous increases in myocardial contractility, glomerular
filtration rate, sodium excretion, urine output, and renal blood flow are
associated most likely with:
A Epinephrine
B Isoproterenol
C Phenylephrine
D Dopamine *
E Norepinephrine
480 IV dopamine (Intropin) properties:
A Promotes renal tubule or solid reabsorption
B Reduces sodium excretion
C Causes reduced ventilatory response to arterial hypoxemia*
D Decreases myocardial contractility
E Increases atrial filling pressures
481 Therapeutic uses for isoproterenol (Isuprel):
A Management of heart block
B Management of severe bradycardia
C Management Torsades de pointes (a ventricular arrhythmia)
D All of the above*
E None of the above; no therapeutic uses for isoproterenol (Isuprel)
482 Properties of dobutamine (Dobutrex):
A Positive inotropic agent; causes significant increase in heart rate
B Promotes catecholamine release
C Mainly acts through dopamine receptors
D Positive inotropic effect is mediated through beta-adrenergic receptor
activation*
E Appropriate for long-term management of myocardial pump-failure
following surgery
483 Examples of beta-2 selective adrenergic agonists
A Metaproterenol
B Terbutaline
C Albuterol
D Metaproterenol; Terbutaline
E Metaproterenol; Terbutaline; Albuterol*
484 Adverse effects associated with beta2 adrenergic receptor agonists:
A Excessive cardiovascular stimulation
B Skeletal muscle tremor
C Over usage of these drugs may predisposed to morbidity immortality
in asthmatics
D Excessive cardiovascular stimulation; Over usage of these drugs may
predisposed to morbidity immortality in asthmatics
E Excessive cardiovascular stimulation; Skeletal muscle tremor; Over
usage of these drugs may predisposed to morbidity immortality in
asthmatics*
485 Alpha-1-selective adrenergic agonists: properties of
A Phenylephrine is an example of the indirect-acting vasoconstrictor
B Metaraminol acts by direct and indirect mechanisms*
C Methoxamine is an indirect acting vasoconstrictor
D Alpha 1-receptor activation decreases calcium influx
E All of the above
486 Clinical use(s) of alpha-1-receptor agonists:
A Management hypotensive states
B Termination of paroxysmal atrial tachycardia
C Nasal decongestant
D Management hypotensive states; Termination of paroxysmal atrial
tachycardia
E Management hypotensive states; Termination of paroxysmal atrial
tachycardia; Nasal decongestant*
487 Primary use for alpha-2-selective adrenergic agonists:
A To manage hypotensive states
B To increase myocardial contractility
C To reduce blood pressure*
D To manage hypotensive states; To increase myocardial contractility
E To manage hypotensive states; To increase myocardial contractility;
To reduce blood pressure
488 Adverse effects associated with clonidine:
A Dry mouth
B Sedation
C Sexual dysfunction
D All of the above*
E Dry mouth; Sedation
489 Clinical uses for sympathomimetic drugs:
A Hypovolemic shock caused by dehydration or blood loss
B Cardiogenic shock (pump failure)
C Cardiac output obstruction
D Loss of peripheral vascular tone
E All of the above*
490 Primary objective sympathomimetic drug use for management of
shock:
A Increase myocardial contractility
B Decreased peripheral resistance
C Promote better renal perfusion
D Ensure adequate CNS perfusion*
E Improve coronary perfusion
491 Most likely to reduce myocardial performance in a damaged heart by
increasing afterload:
A Isoproterenol
B Phenylephrine *
C Low -dose dopamine
D Low -dose epinephrine
E All of the above
492 Receptor system most likely responsible for improved myocardial
contractility when dopamine is administered at low concentrations:
A Muscarinic cholinergic receptors
B Alpha adrenergic receptors
C Beta adrenergic receptors
D Dopamine receptors (D1)*
E Leukotriene receptors
493 Reasonable intervention(s) to reverse cardiogenic shock caused by
acute myocardial infarction
A Supplemental oxygen
B IV nitroglycerin
C Intra -aortic balloon pump
D Revascularization
E All of the above*
494 Phosphodiesterase inhibitor(s) which have positive inotropic actions;
might be used in management of cardiogenic shock
A Phentolamine
B Nitroglycerin
C Amrinone *
D Clonidine
E Caffeine
495 Mechanism by a which methoxamine might terminate paroxysmal
supraventricular tachycardia:
A Direct action at the AV node
B Increases SA nodal rates overdrives the ectopic focus
C Blocks beta-1-receptors
D Causes increased vagal tone through reflex activation secondary to
increase blood pressure*
E Directly inhibit sodium channel conductance
496 Clinical applications of beta-adrenergic antagonists:
A Management of coronary vascular disease
B Treatment of arrhythmias
C Treatment of hypertension
D All the above*
E Management of coronary vascular disease; Treatment of arrhythmias
497 A non-selective beta-adrenergic receptor blocker:
A Metoprolol
B Atenolol
C Timolol *
D Esmolol
E Metoprolol; Atenolol
498 Beta-adrenergic receptor blockers: effects on the heart
A Increase heart rate
B Increased AV nodal refractory period*
C Increased contractility
D Increased myocardial oxygen demand
E Increased phase 4 depolarization
499 Most likely to cause dangerous bronchiolar constriction in asthmatic
patients or patients with COPD
A Metoprolol
B Propranolol *
C Esmolol
D Atenolol
E Metoprolol; Propranolol
500 Mechanism(s) for propranolol-decreased amide local anesthetic
clearance:
A Decreased hepatic blood flow
B Inhibition of hepatic metabolism of local anesthetic
C Both*
D Neither
E All the above
501 The major exception to the rule that additive myocardial depression
between anesthetics and beta-adrenergic antagonists is not excessive:
A Metoprolol
B Propranolol
C Esmolol
D Timolol *
E Nadolol
502 Bradycardia hypotension, refractory to atropine, may occur during
anesthesia in pediatric and adult patients receiving this beta adrenergic receptor
antagonist:
A Nadolol
B Propranolol
C Esmolol
D Timolol *
E All the above
503 Greatest additive cardiovascular effects with inhaled anesthetics in the
presence of beta-adrenergic receptor blockade:
A Enflurane *
B Halothane
C Isoflurane
D All the above
E Enflurane; Halothane
504 Preferred beta-blocking agent to prevent systolic blood pressure
increases associated with direct laryngoscopy in tracheal intubation:
A IV propranolol
B IV labetalol
C V esmolol *
D IV timolol
E IV nadolol
505 Beta-blocker having special benefits for patients undergoing
noncardiac surgery but having significant underlying coronary artery disease--
given IV perioperatively and orally during remainder of hospital stay.
A Esmolol
B Atenolol *
C Propranolol
D Nadolol
E Esmolol; Atenolol
506 Sympathetic Nervous System
A Ganglionic neurotransmitter: acetylcholine
B Generalized response upon sympathetic activation
C Thoraco -lumbar origin for preganglionic cell bodies
D A, B and C*
E All the above
507 Choline ester most susceptible to hydrolysis by acetylcholinesterase:
A Carbachol
B Acetylcholine*
C Methacholine
D Pilocarpine
E All the above
508 Resistant to hydrolysis by acetylcholinesterase
A Carbachol *
B Methacholine
C Both
D Neither
E All the above
509 The highest nicotinic receptor activity among choline esters has:
A Acetylcholine
B Atropine
C Methacholine
D Carbacholine*
E All the above
510 Indicate the direct effects associated with parasympathetic activation:
A Increase heart rate
B Decreased GI motility
C Decrease cardiac contractility*
D Urinary retention
E All the above
511 Bronchoconstriction in an asthmatic:
A Metoprolol *
B Atropine sulfas
C Albuterol
D Ipratropium
E Metoprolol; Atropine sulfas
512 Antimuscarinic drug with highest CNS activity
A Atropine sulfas
B Scopolamine *
C Homatropine
D Muscarine
E Atropine sulfas; Scopolamine
513 Muscarinic agent: enhances transmission through the A-V node:
A Isoproterenol
B Atropine*
C Propranolol
D Methacholine
E Isoproterenol; Atropine
514 Least likely to be used as a mydriatic because of long-duration of
action:
A Homatropine
B Atropine sulfas*
C Cyclopentolate
D Benztropine
E Homatropine; Atropine sulfas
515 Clinically-used to treat sinus bradycardia secondary to acute
myocardial infarction:
A Homatropine
B Atropine sulfas*
C Benztropine
D Tropicamide
E Homatropine; Atropine sulfas
516 Symptoms following DFP exposure (diisopropylfluorophosphate, an
organophosphate poison):
A Constipation
B Salivation*
C Decreased gastric acid secretion
D None of the above
E All the above
517 Location(s) of cholinergic synaptic sites:
A Neuromuscular junction
B Autonomic effector sites innervated by post-ganglionic sympathetic
fibers
C Some CNS synapses
D Neuromuscular junction; Some CNS synapses*
E Autonomic effector sites innervated by post-ganglionic sympathetic
fibers; Some CNS synapses
518 Factors that limit CNS effects of systemic acetylcholine:
administration:
A Poor CNS penetration
B Inactivation by plasma butrylcholinesterase
C Both*
D Neither
E All the above
519 Localization of muscarinic cholinergic receptors:
A Postganglionic parasympathetic effector sites
B Autonomic ganglia cells
C Adrenal medulla
D Postganglionic parasympathetic effector sites; Adrenal medulla
E Postganglionic parasympathetic effector sites; Autonomic ganglia
cells; Adrenal medulla*
520 Highly sensitive to the action of acetylcholinesterase:
A Carbachol
B Bethanechol
C Acetylcholine*
D Carbachol; Acetylcholine
E Carbachol; Acetylcholine; Bethanechol
521 Muscarinic receptor subtype primarily associated with the heart:
A M1
B M2*
C M3
D M4
E M1; M3
522 Most likely to be effective in blocking all ganglionic
neurotransmission:
A Tubocurarine
B Mecamylamine *
C Atropine sulfas
D All of the above
E Tubocurarine; Atropine sulfas
523 Cardiac muscarinic Type M2-receptor mediated action(s):
A Increased phase 4 depolarization rate
B Increased AV nodal conduction velocity
C Decreased atrial and ventricular contractility*
D All the above
E Increased phase 4 depolarization rate; Decreased atrial and ventricular
contractility
524 Ligand-gated ion channels:
A Nicotinic*
B Muscarinic
C Both
D Neither
E None of the above
525 Indirect-acting cholinomimetic:
A Atropine sulfas
B Edrophonium
C Carbachol*
D Acetylcholine
E Ephedrine
526 Cholinergic-receptor-mediated vasodilation -- changes in intracellular
concentration of this ion is principally responsible:
A Sodium
B Potassium
C Chloride
D Calcium*
E Magnesium
527 Cholinergic-mediated vasodilation involves liberation of this
substance, a gas, from endothelial cells:
A Prostaglandins
B Leukotrienes
C Nitric oxide*
D Calcium
E Prostaglandins; Nitric oxide
528 Mechanism(s) of vasodilation mediated by the cholinergic system:
A Cholinergic activation promotes nitric oxide release from endothelial
cells
B Acetylcholine inhibits norepinephrine release from postganglionic
sympathetic fibers
C Both*
D Neither
E All the above
529 Parasympathetic system: negative chronotropic effect --
A Mediated by M2 muscarinic receptors
B Associated with increased diastolic depolarization (increased phase 4
depolarization)
C Both
D Neither*
E All the above
530 Major mechanism responsible for decreased AV nodal conduction
following increased vagal tone:
A Decreased calcium currents in the AV node*
B Secondary affected to reduced norepinephrine release
C Decreased sodium currents in the AV node
D Increased potassium conductance in the AV nodal fibers
E All of the above
531 Associated with excessive vagal tone:
A Partial heart block
B Total heart block
C Other bradyarrhythmias
D All the above*
E Partial heart block; Other bradyarrhythmias
532 Concerning negative inotropism associated with increased vagal tone:
A More prominent in atrial compared to ventricular muscle
B Due to a decrease in inward calcium currents
C Both*
D Neither
E None of the above
533 Mechanisms by which muscarinic stimulation reduces ventricular
contractility:
A Reduces ventricular responds to norepinephrine
B Reduces norepinephrine release from adrenergic terminals
C Both*
D Neither
E Reduces ventricular responds to norepinephrine; Both
534 Effects of muscarinic receptor activation and cardiac ionic currents:
A Decreases potassium currents in atrial muscle and in SA nodal MAb
nodal tissue
B Increases in slow, inward calcium currents
C Decreased in diastolic depolarization (decrease in phase 4
depolarization)*
D Decreases potassium currents in atrial muscle and in SA nodal MAb
nodal tissue; Decreased in diastolic depolarization (decrease in phase 4
depolarization)
E Increases in slow, inward calcium currents; Decreased in diastolic
depolarization (decrease in phase 4 depolarization)
535 Effect(s) of muscarinic agonists on the gastrointestinal and urinary
tracts:
A Increased intestinal peristalsis
B Increased tone
C Increased contraction amplitude
D Increase ureteral peristalsis
E All the above*
536 Substances that increase nitric oxide production:
A Substance P
B Bradykinin
C Acetylcholine
D Substance P; Bradykinin
E Substance P; Bradykinin; Acetylcholine*
537 Clinical uses of bethanecol:
A Management of postoperative abdominal distention
B Management of esophageal reflux
C Postoperative urinary bladder stimulant
D Treatment of reduced salivation secondary to radiation therapy
E All the above*
538 Opthalmological uses of cholinomimetics:
A Acetylcholine may be used as a miotic
B Treatment of glaucoma
C Used along with mydriatic agent in breaking iris-lens adhesions
D Acetylcholine may be used as a miotic; Treatment of glaucoma
E Acetylcholine may be used as a miotic; Treatment of glaucoma; Used
along with mydriatic agent in breaking iris-lens adhesions*
539 Major contraindications -- muscarinic agonists
A Asthma
B Hyperthyroidism
C Peptic ulcer
D Coronary vascular disease
E All the above*
540 Cholinergic-receptor-mediated vasodilation -- changes in intracellular
concentration of this ion is principally responsible:
A Sodium
B Potassium
C Chloride
D Calcium*
E Magnesium
541 Cholinergic-mediated vasodilation involves liberation of this
substance, a gas, from endothelial cells:
A Prostaglandins
B Leukotrienes
C Nitric oxide*
D Calcium
E Prostaglandins; Nitric oxide
542 Mechanism(s) of vasodilation mediated by the cholinergic system:
A Cholinergic activation promotes nitric oxide release from endothelial
cells
B Acetylcholine inhibits norepinephrine release from postganglionic
sympathetic fibers
C Both*
D Neither
E All the above
543 Parasympathetic system: negative chronotropic effect --
A Mediated by M2 muscarinic receptors
B Associated with increased diastolic depolarization (increased phase 4
depolarization)
C Both
D Neither*
E All the above
544 Major mechanism responsible for decreased AV nodal conduction
following increased vagal tone:
A Decreased calcium currents in the AV node*
B Secondary affected to reduced norepinephrine release
C Decreased sodium currents in the AV node
D Increased potassium conductance in the AV nodal fibers
E All of the above
545 Associated with excessive vagal tone:
A Partial heart block
B Total heart block
C Other bradyarrhythmias
D All the above*
E Partial heart block; Other bradyarrhythmias
546 Concerning negative inotropism associated with increased vagal tone:
A More prominent in atrial compared to ventricular muscle
B Due to a decrease in inward calcium currents
C Both*
D Neither
E All the above
547 Mechanisms by which muscarinic stimulation reduces ventricular
contractility:
A Reduces ventricular responds to norepinephrine
B Reduces norepinephrine release from adrenergic terminals
C Both*
D Neither
E All the above
548 Effects of muscarinic receptor activation and cardiac ionic currents:
A Decreases potassium currents in atrial muscle and in SA nodal MAb
nodal tissue
B Increases in slow, inward calcium currents
C Decreased in diastolic depolarization (decrease in phase 4
depolarization)*
D Decreases potassium currents in atrial muscle and in SA nodal MAb
nodal tissue; Decreased in diastolic depolarization (decrease in phase 4
depolarization)
E Increases in slow, inward calcium currents; Decreased in diastolic
depolarization (decrease in phase 4 depolarization)
549 Effect(s) of muscarinic agonists on the gastrointestinal and urinary
tracts:
A Increased intestinal peristalsis
B Increased tone
C Increased contraction amplitude
D Increase ureteral peristalsis
E All the above*
550 Substances that increase nitric oxide production:
A Substance P
B Bradykinin
C Acetylcholine
D Substance P; Bradykinin
E Substance P; Bradykinin; Acetylcholine*
551 Clinical uses of bethanecol:
A Management of postoperative abdominal distention
B Management of esophageal reflux
C Postoperative urinary bladder stimulant
D Treatment of reduced salivation secondary to radiation therapy
E All the above*
552 Opthalmological uses of cholinomimetics:
A Acetylcholine may be used as a miotic
B Treatmen t of glaucoma
C Used along with mydriatic agent in breaking iris-lens adhesions
D All the above
E Acetylcholine may be used as a miotic; Treatmen t of glaucoma; Used
along with mydriatic agent in breaking iris-lens adhesions*
553 Major contraindications -- muscarinic agonists
A Asthma
B Hyperthyroidism
C Peptic ulcer
D Coronary vascular disease
E All the above*
554 Classes of anticholinesterase drugs:
A Reversible , short-acting
B Intermediate , carbamylating
C long-acting , phosphorylate agents
D All the above
E Reversible , short-acting; Intermediate , carbamylating; long-acting ,
phosphorylate agents*
555 anticholinesterase agent; quaternary ammonium compound;
intermediate-duration, carbamylating agent:
A physostigmine
B Neostigmine *
C Edrophonium
D Tacrine
E Atropine sulfas
556 Renal clearance -- acetylcholinesterase inhibitors--
A Actively secreted into renal tubule lumen
B Renal clearance: 50% for neostigmine
C Renal clearance: 75% for edrophonium and pyridostigmine
D All of the above*
E Actively secreted into renal tubule lumen; Renal clearance: 50% for
neostigmine
557 In organophosphate poisoning, this agent may be capable of
reactivating inhibited acetylcholinesterase:
A Atropine sulfas
B Pilocarpine
C Mecamylamine
D 2-PAM*
E All of the above
558 Consequences of acetylcholinesterase inhibitor application to the
conjunctiva
A Relaxation of the pupillary sphincter muscle
B Relaxation of the ciliary muscle
C Both
D Neither*
E Relaxation of the pupillary sphincter muscle; Relaxation of the
ciliary muscle
559 Types of glaucoma:
A Primary
B Secondary
C Congenital
D All the above*
E Primary; Secondary
560 Anticholinesterases: used in treating glaucoma--
A Echothiophate
B Demecarium
C Atropine sulfas
D Echothiophate & Demecarium *
E All the above
561 Probable cause of myasthenia gravis:
A Excessive synthesis of cholinergic receptors
B Inadequate synthesis of acetylcholine
C Failure of acetylcholine reuptake system
D Binding of anti--muscarinic receptor antibodies to the muscarinic
cholinergic receptor
E Binding of anti-nicotinic receptor antibodies to the nicotinic
cholinergic receptor*
562 Rationale for prescribing anticholinesterase drugs to patients with
myasthenia gravis:
A Increase acetylcholine turnover
B Increase receptor number
C Increase amount of acetylcholine available of neuromuscular
junctions*
D Reduce choline reuptake
E All of the above
563 Associated disorders in myasthenic patients --
A Thymic abnormalities
B Hyperthyroidism
C Other autoimmune disorders
D Ventilatory dysfunction
E All the above*
564 General clinical uses: anticholinesterases
A Antagonist -assisted reversal of neuromuscular blockade produced by
nondepolarizing neuromuscular-blocking drugs
B Myasthenia gravis management
C Glaucoma treatment
D Treatment of paralytic ileus and urinary bladder atony
E All of the above*
565 Drugs used for antagonist-assisted neuromuscular-blockade reversal
A Acetylcholine
B Physostigmine
C DFP
D Edrophonium *
E All of the above
566 Determination of the recovery rates from neuromuscular-blockade
when antagonist-assisted reversal is used:
A Spontaneous recovery rate from the blocking drug
B Activity of the pharmacologic antagonist (anticholinesterase drug)*
C Both (sum of A plus B)
D Differenc (A- B)
E All of the above
567 When our anticholinesterase agents usually administered to enhance
neuromuscular blockade reversal?
A Before the neuromuscular-blocking drug is given
B While the neuromuscular-blocking drug is being infused
C During the spontaneous neuromuscular-blockade recovery, following
cessation of neuromuscular-blocking administration*
D Before the neuromuscular-blocking drug is given; While the
neuromuscular-blocking drug is being infused
E All of the above
568 Pharmacologic antagonism (anticholinesterase drugs) would likely be
more effective for which type of neuromuscular blocking drug?
A Short -or intermediate-acting neuromuscular-blocking drugs*
B Long -acting nondepolarizing neuromuscular-blockade
C Equally effective
D All of the above
E Short -or intermediate-acting neuromuscular-blocking drugs; Long
-acting nondepolarizing neuromuscular-blockade
569 Rationale for using muscarinic antagonists in pharmacologic
(anticholinesterase-mediated) reversal of neuromuscular-blockade:
A Increases acetylcholine concentration that neuromuscular junctions
B Inhibits acetylcholinesterase
C Minimizes muscarinic receptor-mediated effects of anticholinesterase
drugs*
D Increases acetylcholine concentration that neuromuscular junctions;
Inhibits acetylcholinesterase
E All of the above
570 which antimuscarinic agent might be used in combination with an
anticholinesterase when desiring reversal of neuromuscular-blockade and
opioid-based maintenance anesthesia has been used:
A Edrophonium
B High -dose atropine (10-15 ug/kg)*
C Neostigmine
D All of the above
E Edrophonium; High -dose atropine (10-15 ug/kg)
571 More effective in reversing deep neuromuscular-blockade produced
by continuous atracurium (Tracrium), vecuronium (Norcuron), or pancuronium
(Pavulon) infusions
A Edrophonium
B Neostigmine *
C Both are equally effective
D Edrophonium; Neostigmine
E All of the above
572 Factor(s) that may prevent or inhibit anticholinesterase-mediated
antagonism of neuromuscular-blockade:
A Hyperthermia
B Respiratory alkalosis
C Hyperkalemia
D Certain antibiotics*
E All of the above
573 Reversal of phase II block (following prolonged repeated
succinylcholine (Anectine)) in patients with normal plasma cholinesterase:
A Edrophonium
B Neostigmine
C Both*
D Neither
E All of the above
574 Current primary therapeutic rationale for using anticholinergic
preoperative medication:
A Sedation
B Antisialagogue effects
C Both*
D Neither
E All of the above
575 In using anticholinergic drugs as preoperative medication in a patient
with glaucoma: drug least likely to have an effect on pupil size
A Scopolamine bromid
B Atropine sulfas
C Glycopyrrolate *
D All of the above
E Scopolamine bromid; Atropine sulfas
576 Preferred anticholinergic drug when sedation is the principal
objective, preoperatively:
A Atropine sulfas
B Glycopyrrolate
C Scopolamine bromid*
D Atropine sulfas; Glycopyrrolate
E All of the above
577 Atropine: most likely to increase heart rate in this patient population:
A Young adult*
B Infants
C Elderly
D All of the above
E Young adult; Infants
578 Anticholinergic drug most likely to be used clinically to promote
bronchodilation:
A IV atropine
B Aerosolized atropine
C Aerosolized ipratropium bromide *
D Scopolamine bromid
E All of the above
579 More effective in producing bronchodilation in patients with chronic
bronchitis or emphysema:
A Ventolin,Proventil (beta-adrenergic agonist)
B Ipratropium (Atrovent) (antimuscarinic agent)*
C Equally effective
D Ventolin,Proventil (beta-adrenergic agonist); Ipratropium (Atrovent)
(antimuscarinic agent)
E All of the above
580 Atropine is effective in blocking reflex cardiac slowing secondary to:
A Carotid sinus stimulation
B Pressure on the eyeballs
C Peritoneal stimulation which may occur or during surgery
D Carotid sinus stimulation; Pressure on the eyeballs
E Carotid sinus stimulation; Pressure on the eyeballs; Peritoneal
stimulation which may occur or during surgery*
581 Benzodiazepine pharmacology is most closely linked to which
neurotransmitter system?
A Noradrenergic
B GABA system*
C Cholinergic
D Glycine system
E All of the above
582 Therapeutic disadvantage(s) of benzodiazepines:
A Amestic effects
B High cost
C Psychologic dependencies
D All of the above*
E Amestic effects; High cost
583 Pharmacological basis for prescribing benzodiazepines for anxiety:
A Low risk of drug interactions based on hepatic enzyme induction
B High therapeutic index
C Availability of flumazenil (Romazicon) to manage overdosage
D All of the above*
E Low risk of drug interactions based on hepatic enzyme induction;
High therapeutic index
584 Ion channel that contains the GABA receptor:
A Sodium
B Calcium
C Chloride*
D Potassium
E Sodium; Calcium
585 Most useful in reversing symptoms of benzodiazepine overdosage:
A Amphetamine
B Buspirone
C Flumazenil *
D Naltrexone
E Amphetamine; Buspirone
586 A comatose patient is brought to the emergency department with
severe respiratory depression caused by diazepam overdosage. Reasonable
intervention at this point include:
A Administer naloxone to block the drug's effect at the receptor
B Provide supportive therapy until the drug effect wears off.
C Administer flumazenil
D Provide supportive therapy until the drug effect wears off ;
Administer flumazenil*
E All of the above
587 Pupillary constriction:
A Diazepam
B Buspirone *
C Triazolam
D Flumazenil
E Diazepam; Buspiron
588 Short-acting benzodiazepine
A Diazepam
B Flurazepam
C Triazolam *
D Buspirone
E All of the above
589 May be especially effective in treating anxiety associated with
nicotine withdrawal:
A Meprobamate
B Clonidine *
C Both
D Neither
E All of the above
590 Anxiolytic effects
A Diazepam
B Clonidine
C Both*
D Neither
E All of the above
591 Benzodiazepine most appropriate in patients with impaired hepatic
function:
A Lorazepam because of first-pass effects and because it lacks long-
acting metabolites.
B Oxazepam since it has no long-acting metabolites and is conjugated to
an inactive glucuronide directly.*
C Buspirone since it has limited abuse potential
D All of the above
E Lorazepam because of first-pass effects and because it lacks long-
acting metabolites; Oxazepam since it has no long-acting metabolites and is
conjugated to an inactive glucuronide directly
592 Anxiolytic drug acting through serotonin receptors:
A Diazepam
B Buspirone *
C Triazolam
D Phenobarbital
E Diazepam; Buspirone
593 A chronic alcohol and diazepam abuser is hospitalized for
management of withdrawal symptoms. Buspirone, an anxiolytic, is
administered to alleviate withdrawal, but the symptoms continue and worsen.
Why?
A Buspirone dosage too low.
B Buspirone does not exhibit cross-tolerance with sedative-
hypnotic/benzodiazepines.*
C Management of alcohol or benzodiazepine withdrawal must be
managed with a stronger drug, such as haloperidol.
D Buspirone is too rapidly inactivated (first-pass effect) to be useful in
managing withdrawal.
E Buspirone dosage too low; Buspirone does not exhibit cross-tolerance
with sedative-hypnotic/benzodiazepines
594 Antianxiety drug(s)
A Propranolol
B Buspirone
C Phenobarbital
D All of the above*
E Propranolol; Buspirone
595 Pharmacology effect of Zolpidem (Ambien):
A Antagonized by flumazenil*
B Muscle relaxation
C Minimal hypnotic properties
D Long duration of action
E Antagonized by flumazenil; Muscle relaxation
596 Contraindicated in patients with a history of acute intermittent
porphyria:
A Diazepam
B Phenobarbital *
C Clonazepam
D Buspirone
E Diazepam; Phenobarbital
597 Indicate the i.v. anesthetic used for induction.
A Methohexital
B Thiopental *
C Both
D Neither
E Diazepam
598 May be especially effective in treating anxiety associated with
nicotine withdrawal:
A Meprobamate
B Clonidine *
C Both
D Neither
E All of the above
599 Therapeutic disadvantage(s) of benzodiazepines:
A Amestic effects
B High cost
C Psychologic dependencies
D All of the above*
E Neither
600 i.v. anesthetic used for induction.
A Methohexital
B Thiopental *
C Both
D Neither
E All of the above
601 A chronic alcohol and diazepam abuser is hospitalized for
management of withdrawal symptoms. Buspirone, an anxiolytic, is
administered to alleviate withdrawal, but the symptoms continue and worsen.
Why?
A Buspirone dosage too low.
B Buspirone does not exhibit cross-tolerance with sedative-
hypnotic/benzodiazepines.*
C Management of alcohol or benzodiazepine withdrawal must be
managed with a stronger drug, such as haloperidol.
D Buspirone is too rapidly inactivated (first-pass effect) to be useful in
managing withdrawal.
E All of the above
602 Anxiolytic effects
A Diazepam
B Clonidine
C Both*
D Neither
E All of the above
603 Contraindicated in patients with a history of acute intermittent
porphyria:
A Diazepam
B Phenobarbital *
C Clonazepam
D Buspirone
E All of the above
604 Zolpidem:
A Antagonized by flumazenil *
B Muscle relaxation
C Minimal hypnotic properties
D Long duration of action
E All of the above
605 A comatose patient is brought to the emergency department with
severe respiratory depression caused by diazepam overdosage. Reasonable
intervention at this point include:
A Administer naloxone to block the drug's effect at the receptor
B Provide supportive therapy until the drug effect wears off.
C Administer flumazenil
D Provide supportive therapy until the drug effect wears off ; Administer
flumazenil*
E All of the above
606 Benzodiazepine pharmacology is most closely linked to which
neurotransmitter system?
A Noradrenergic
B GABA system*
C Cholinergic
D Glycine system
E All of the above
607 Pharmacological basis for prescribing benzodiazepines for anxiety:
A Low risk of drug interactions based on hepatic enzyme induction
B High therapeutic index
C Availability of flumazenil to manage overdosage
D All of the above*
E All of the above
608 Antianxiety drug(s)
A Propranolol
B Buspirone
C Phenobarbital
D All of the above*
E All of the above
609 Most useful in reversing symptoms of benzodiazepine overdosage:
A Amphetamine
B Buspirone*
C Flumazenil
D Naltrexone
E All of the above
610 Anxiolytic drug acting through serotonin receptors:
A Diazepam
B Buspirone
C Triazolam *
D Phenobarbital
E All of the above
611 Pupillary constriction:
A Diazepam
B Buspirone*
C Triazolam
D Flumazenil
E All of the above
612 Ion channel that contains the GABA receptor:
A Sodium
B Calcium
C Chloride*
D Potassium
E All of the above
613 Short-acting benzodiazepine
A Diazepam
B Flurazepam
C Triazolam *
D Buspirone
E All of the above
614 Neuroendocrine effects of morphine
A Decrease in corticotropin releasing factor
B Decrease in gonadotropin-releasing factor
C Decrease in cortisol levels
D Decrease in corticotropin releasing factor; Decrease in gonadotropin-
releasing factor; Decrease in cortisol levels*
E All of the above
615 Narcotic agonists: most serious adverse effect:
A Cardiac arrhythmias
B Respiratory depression*
C Convulsions
D Endogenous depression
E All of the above
616 Physiological effects of meperidine:
A Miosis
B Cns depressive effects due to metabolite.
C Nausea
D Miosis; Nausea*
E All of the above
617 An opioid with both agonist and antagonist properties:
A Morphine
B Meperidine
C Pentazocine *
D Oxycodone
E All of the above
618 Physiological effects of morphine:
A Chronic diahrrea
B Mydriasis
C Respiratory depression*
D Chronic diahrrea; Respiratory depression
E All of the above
619 Opioid direct action on neurons:
A May close a voltage-gated calcium channel on presynaptic nerve
terminals, resulting in reduced transmitter release*
B Depolarization of postsynaptic neurons by opening a sodium channel
C Blockade of sodium channels, in a manner similar to the neurotoxin,
tetrodotoxin
D May close a voltage-gated calcium channel on presynaptic nerve
terminals, resulting in reduced transmitter release; Blockade of sodium
channels, in a manner similar to the neurotoxin, tetrodotoxin
E Depolarization of postsynaptic neurons by opening a sodium
channel; Blockade of sodium channels, in a manner similar to the
neurotoxin, tetrodotoxin
620 Opioid effect least likely to exhibit tolerance following prolonged
opioid administration:
A Analgesia
B Bradycardia
C Respiratory depression
D Miosis*
E Euphoria
621 Very short-acting; substrate for tissue esterases
A Morphine
B Levorphanol
C Remifentanil *
D Fentanyl
E None of the above
622 Crosses the blood-brain barrier most easily:
A Morphine
B Codeine*
C Both
D Neither
E All of the above
623 Anatomical sites of opioid action: pain-modulating descending
pathways --
A Rostral ventral medulla
B Locus ceruleus
C Periaqueductal gray
D Rostral ventral medulla; Periaqueductal gray
E Rostral ventral medulla; Periaqueductal gray; Periaqueductal gray*
624 Opioid + Route of Administration most likely associated with truncal
rigidity:
A Oral pentazocine
B Oral propoxyphene
C Intravenous, rapidly infused, alfentanil*
D Intravenous naloxone
E Intravenous naltrexone
625 Most reliable indicator of opioid-mediated respiratory depression:
A Reduced patient responsiveness
B Alveolar PCO2 increased
C Depressed patient response to a carbon dioxide challenge*
D Increased levels of alkaline phosphatase
E Increased levels of creatinine
626 Principal alkaloid in opium (derived from opium poppy)
A Morphine*
B Fentanyl
C Codeine
D Naloxone
E Meperidine
627 Descriptive of the drug that may have an agonist effect at one opioid
receptor site and an antagonist effect at another opioid receptor site.
A Full agonist
B Pure antagonist
C Mixed agonist-antagonist*
D Endorphin
E Dynorphins
628 Crosses the neonatal blood-brain barrier:
A Morphine
B Meperidine
C Fentanyl
D Oxycodone
E All of the above*
629 Main tissue reservoir for opioids:
A Fat
B Skeletal muscle*
C Spleen
D Liver
E Lungs
630 Polar glucuronide metabolite of this opioid may have higher analgesic
potency then the parent compound:
A Remifentanil
B Morphine*
C Both
D Neither
E All of the above
631 Effective antitussive(s)
A Codeine
B Morphine
C Dextromethorphan
D oxycodone
E All the above*
632 Opioid antagonist with shortest duration of action:
A Naloxone *
B Nalmefene
C Naltrexone
D Al l about equal duration of action
E These agents are not opioid antagonists
633 Signs and symptoms of acute opioid withdrawal (abstinence
syndrome)
A Yawning
B Lacrimation
C Hyperventilation
D Diarrhea
E All the above*
634 Physiological effect NOT typically associated with opioids:
A Constipation
B Constriction of biliary smooth muscle
C Enhanced renal function*
D Limited direct cardiovascular effects
E May prolong labor
635 Contraindications/caution for opioid use:
A Patients with Addison's disease
B Patients with impaired pulmonary function
C Patients with head injury
D Patients with impaired pulmonary function; Patients with head
injury
E Patients with Addison's disease; Patients with impaired pulmonary
function; Patients with head injury*
636 Useful drug(s) for opioid detoxification and maintenance of chronic
relapsing heroin addict:
A Methadone
B Levomethadyl acetate
C Propoxyphene
D Methadone; Levomethadyl acetate*
E Methadone; Levomethadyl acetate; Propoxyphene
637 Mixed agonist-antagonist:
A Naloxone
B Naltrexone
C Hydromorphone
D Nalbuphine *
E Diphenoxylate
638 Routes of opioid agonist administration:
A Transdermal
B Patient -controlled analgesia (PCA)
C Intranasal
D Transdermal; Patient -controlled analgesia (PCA)
E Transdermal; Patient -controlled analgesia (PCA); Intranasal*
639 Used in treatment diarrhea:
A Difenoxin
B Diphenoxylate
C Loperamide
D Diphenoxylate; Loperamide
E Difenoxin; Diphenoxylate; Loperamide*
640 Neurolept analgesia:
A Propofol
B Droperidol + fentanyl citrate*
C Ketamine
D Enflurane + hydoxyzine
E All of the above
641 Unpredictable hepatitis occurrence has been one factor that reduce the
use of this inhalational anesthetic:
A Isoflurane
B Enflurane
C Halothane*
D Desflurane
E All of the above
642 Properties of inhalational anesthetics:
A More soluble in blood leads to more rapid induction.
B Anesthetic tension in the blood rises more rapidly for highly soluble
drugs.
C Brain concentration of nitrous oxide rises rapidly.*
D Rate of pulmonary blood flow significantly affects time to anesthetic
equilibrium.
E All of the above
643 Accepted measure of anesthetic potency.
A Lipid solubility
B Speed of induction.
C Presence of a "second gas effect"
D Minimum alveolar concentration (MAC value)*
E All of the above
644 Very brief anesthetic effects of this barbiturate is explained by "rapid
redistribution":
A Nitrous oxide
B Halothane
C Thiopental*
D Fentanyl
E All of the above
645 Fastest rate of rise of arterial gas tension (as% of inspired tension):
A Halothane
B Ether
C Nitrous oxide*
D Isoflurane
E All are equal
646 Factors determining anesthetic partial pressure in arterial blood:
A Anesthetic concentration in the inspired air
B Pulmonary ventilation
C Transfer of gas from blood to body tissues
D Anesthetic concentration in the inspired air; Transfer of gas from
blood to body tissues
E Anesthetic concentration in the inspired air; Pulmonary ventilation;
Transfer of gas from blood to body tissues*
647 Anesthetic solubility in the blood:
A The more soluble in age and is in the blood the less must be
dissolved to raise its partial pressure
B Inhalational agent solubility is a relatively unimportant factor in
determining the speed of induction recovery
C Induction times may be prolonged with methoxyflurane because of
its high solubility*
D All of the above
E Neither
648 Depth of anesthesia primarily depends on:
A Patient age
B Rate of hepatic anesthetic metabolism
C Anesthetic partial pressure in brain*
D Rate of transfer of gas from the blood to alveoli
E Rate of redistribution of the anesthetic from the brain to other tissues
649 Factors that determine how quickly anesthetics pass from the inspired
gas to the blood:
A Anesthetic solubility in the blood
B Partial pressures in arterial and mixed venous blood
C Pulmonary blood flow
D Anesthetic solubility in the blood; Pulmonary blood flow
E Anesthetic solubility in the blood; Partial pressures in arterial and
mixed venous blood; Pulmonary blood flow*
650 Pulmonary blood flow & gas transfer from alveoli to blood
A Pulmonary blood flow is inversely related to cardiac output
B With higher pulmonary flow, there is an initial relative decrease in
the rate of rise of anesthetic tension*
C Both
D Neither
E All of the above
651 Factors affecting transport of anesthetic from arterial blood to tissues:
A Rate of delivery of anesthetic to tissue
B Solubility of the gas in tissues
C Difference impartial pressures between arterial blood and tissue
D Rate of delivery of anesthetic to tissue; Difference impartial
pressures between arterial blood and tissue
E Rate of delivery of anesthetic to tissue; Solubility of the gas in
tissues; Difference impartial pressures between arterial blood and tissue*
652 Sevoflurane :
A Low blood solubility
B Resembles desflurane in pharmacological properties
C Very commonly used
D Low blood solubility; Very commonly used
E Low blood solubility; Resembles desflurane in pharmacological
properties;Very commonly used*
653 Nitrous oxide advantages:
A Excellent analgesia
B Nonflammable
C Very rapid onset and recovery
D Excellent analgesia; Very rapid onset and recovery
E Excellent analgesia; Nonflammable; Very rapid onset and recovery*
654 Anesthetic potency --
A At equilibrium the partial pressure of the anesthetic gas in the lung is
usually significantly higher than the partial pressure of the gas in the brain.
B Concerning anesthetic gases -- there is a rapid equilibrium between
blood and brain partial pressures*
C MAC values are highly dependent on the nature of the painful
stimulus
D MAC values are relatively sensitive to patient gender, height, weight,
and anesthesia duration
E All of the above
655 Concerning measurement of anesthetic potency:
A Anesthetic potency is determined by measuring brain anesthetic
concentration
B Anesthetic potency is determined based on alveolar gas
concentration*
C Both
D Neither
E All of the above
656 Therapeutics-index range -- general anesthetics:
A 0.5-1
B 2-4*
C 5-10
D > 10
E All of the above
657 Nitrous oxide:
A Effective muscle relaxation
B Should not be used in the patient has occluded middle ear*
C Significant depression of myocardial contractility
D Significant respiratory depression
E Effective muscle relaxation; Should not be used in the patient has
occluded middle ear; Significant depression of myocardial contractility
658 Apnea, non-reacted dilated pupils, circulatory failure, and
hypertension are most likely associated with this stage of general anesthesia:
A Amnesia /analgesia
B Delirium
C Surgical anesthesia
D Medullary depression*
E All of the above
659 Isoflurane (Forane) anesthesia --
A Rapid , smooth adjustment of depth of anesthesia with limited effects
on pulse or respiration
B Easily controlled depth of anesthesia
C Significant renal toxicity
D Rapid , smooth adjustment of depth of anesthesia with limited effects
on pulse or respiration; Easily controlled depth of anesthesia*
E Rapid , smooth adjustment of depth of anesthesia with limited effects
on pulse or respiration; Easily controlled depth of anesthesia; Significant
renal toxicity
660 Enflurane (Ethrane)--
A Widely used in pediatric cases
B May cause concern if patient has a seizure-disorder history*
C Skeletal muscle relaxation:inadequate for surgery
D Difficult adjustment of anesthesia depth due to significant effects on
pulse and respiration
E All of the above
661 Central gaze, constricted pupils, regular aspirations, and the anesthetic
depth is sufficient that noxious stimuli does not cause reflexes or excessive
autonomic effects
A Analgesia /amnesia
B Delirium
C Surgical anesthesia*
D Medullary depression
E All of the above
662 Desflurane used for which tipe of anesthesia:
A Associated with laryngospasm and coughing*
B Relatively high solubility
C Typically provides adequate muscle relaxation
D Malignant hyperthermia may be a problem, as with enflurane
(Ethrane)
E All of the above
663 Nitrous oxide anesthesia:
A Relativity solubility of nitrous oxide prevents rapid absorption of
significant quantities of gas
B May be associated with diffusional hypoxia
C Nitrous oxide is typically used in combination with the inducing
agent (e.g. thiopental , the skeletal muscle relaxant, and hyperventilation
D May be associated with diffusional hypoxia; Nitrous oxide is
typically used in combination with the inducing agent (e.g. thiopental , the
skeletal muscle relaxant, and hyperventilation*
E Relativity solubility of nitrous oxide prevents rapid absorption of
significant quantities of gas; May be associated with diffusional hypoxia
664 Associated with malignant hyperthermia
A Halothane
B Isoflurane
C Both*
D Neither
E All of the above
665 Nitrous oxide disadvantages
A No skeletal muscle relaxation
B Weak anesthetic
C Air pockets in closed spaces expand
D Post -anesthesia hypoxia
E All of the above*
666 Isoflurane :
A Minimal muscle relaxation
B Promotes uterine muscle contraction; desirable if uterine contraction
is required to limit blood loss
C Stimulates airway reflexes -- increasing secretions, coughing, and
laryngospasm
D Convulsive activity is associated with isoflurane, similar to
enflurane*
E All of the above
667 Halothane --
A Usually adequate analgesia
B Likely to provide adequate muscle relaxation
C Commonly used in adults; less likely to be used in children
D Reversible reduction of GFR*
E All of the above
668 Guedel's four stages of general anesthesia:
A Medullary depression
B Surgical anesthesia
C Delirium
D Analgesia /amnesia
E All of the above*
669 IV opioids -- anesthesia:
A Hypotension not associated opioid use
B For certain procedures, large-dose morphine can be used to obtain
anesthesia*
C Both
D Neither
E All of the above
670 Innovar --
A Haloperidol + nitrous oxide
B Chlorpromazine + midazolam
C Droperidol + nitrous oxide
D Fentanyl + meperidine
E Fentanyl + droperidol*
671 Rationale for use of intravenous anesthetic agents with inhalational
agents:
A Enhance or provide analgesia
B Enable lower doses of inhalational agent to be used
C Reduce visceral reflex responses
D Enable lower doses of inhalational agent to be used; Enable lower
doses of inhalational agent to be used
E Enhance or provide analgesia; Enable lower doses of inhalational
agent to be used; Enable lower doses of inhalational agent to be used*
672 Antagonizes droperidol -mediated extrapyramidal side effects
A L-DOPA
B Phentolamine
C Benztropine *
D Propranolol
E None of the above
673 Ketamine --
A Reduced muscle tone
B Significant amnesia*
C Minimal analgesia
D Reduced muscle tone; Minimal analgesia
E Minimal analgesia Reduced muscle tone; Significant amnesia
674 Component(s) of balanced anesthesia may include:
A Muscle relaxants
B Thiopental
C Midazolam
D Muscle relaxants; Midazolam
E Muscle relaxants; Thiopental; Midazolam*
675 Droperidol + fentanyl :
A Neuroleptic analgesia*
B Dissociative anesthesia
C Both
D Neither
E All of the above
676 Rapid recovery time associated with remifentanil use: Cause --
A Fast renal excretion of unchanged drug
B Rapid hepatic, cytochrome P450 metabolism
C Esterase -mediated inactivation*
D Fast renal excretion of unchanged drug; Esterase -mediated
inactivation
E Fast renal excretion of unchanged drug; Esterase -mediated
inactivation; Rapid hepatic, cytochrome P450 metabolism
677 Benzodiazepines: Diazepam pharmacology --
A Advantage : combination with opioids do not result in significant
cardiovascular depression
B Central nervous system depression can be reversed with naltrexone
C Effective analgesic
D Associated with anterograde amnesia*
E All of the above
678 Benzodiazepines in anesthesia: may be used alone for procedures not
requiring analgesic, including
A DC cardioversion
B Some radiological procedures
C Endoscopy
D Cardiac catherization
E All the above*
679 More potent then fentanyl :
A Alfentanil
B Sufentanil *
C Both
D Neither
E All of the above
680 Characteristics of thiopental administration:
A Cough , laryngospasm, bronchospasm
B Significant post-operative excitement or vomiting
C Absolute contraindications in patients with acute intermittent
porphyria or variegate porphyria
D Cough , laryngospasm, bronchospasm; Absolute contraindications in
patients with acute intermittent porphyria or variegate porphyria*
E Cough , laryngospasm, bronchospasm; Significant post-operative
excitement or vomiting; Absolute contraindications in patients with acute
intermittent porphyria or variegate porphyria
681 Diffusional hypoxia
A Dilutes alveolar oxygen; may cause postoperative hypoxia
B Lessened by supplemental oxygen administration
C Both*
D Neither
E All of the above
682 Reverses respiratory depression cause by opioids:
A Naloxone
B Nalmefene
C Naltrexone
D Naloxone; Naltrexone
E Naloxone; Naltrexone; Nalmefene*
683 Compared to morphine -- fentanyl
A Less amnestic
B more likely to provoke hypotensive responses
C Increased duration of respiratory depression
D None of the above*
E All of the above
684 Examples of intravenous opioids -- used in anesthesia
A Remifentanil
B Pentazocine
C Alfentanil
D Remifentanil; Alfentanil *
E Remifentanil; Alfentanil; Pentazocine
685 Example(s) of intravenous benzodiazepines used in anesthesia --
A Diazepam
B Flumazenil
C Midazolam
D Diazepam; Midazolam*
E Diazepam; Midazolam; Flumazenil
686 Thiopental :
A Excellent analgesic
B Rapid recovery (20-30 minutes) due to rapid hepatic metabolism
C Absence of respiratory depression
D Crosses the placental barrier, depressing the fetus*
E Significant cardiovascular effects, except in the presence of
hypovolemia

Você também pode gostar